Sei sulla pagina 1di 153

VIOLETA BAHILIDAD PHILIPPINES

VS

PEOPLE

OF

THE

G.R. No. 185195, March 17, 2010 Malversation of Public Funds Facts: Acting on a complaint filed by a Concerned Citizen of Sarangani Province with the Office of the Ombudsman-Mindanao against Mary Ann Gadian, Amelia Carmela Zoleta, both assigned to the Office of the Vice-Governor, and a certain Sheryll Desiree Tangan, from the Office of the Sangguniang Panlalawigan, for their alleged participation in the scheme of giving fictitious grants and donations using funds of the provincial government, a special audit was conducted in Sarangani province. The Special Audit Team, created for the purpose, conducted its investigation from June 1 to July 31, 2003. Included in the list of alleged fictitious associations that benefited from the financial assistance given to certain NonGovernmental Organizations (NGOs), Peoples Organizations (POs), and Local Governmental Units (LGUs) was Women in Progress (WIP), which received a check in the amount of P20,000.00, issued in the name of herein petitioner Bahilidad, as the Treasurer thereof. Based on its findings, the Special Audit Team recommended the filing of charges of malversation through falsification of public documents against the officials involved. Issue: Is petitioner guilty of malversation of public funds? Ruling: NO. In the instant case, petitioner was found guilty of conspiring with Zoleta and other public officials in the commission of the crime of Malversation of Public Funds through Falsification of Public Documents. The trial

court relied on the dictum that the act of one is the act of all. It is necessary that a conspirator should have performed some overt act as a direct or indirect contribution to the execution of the crime committed. The overt act may consist of active participation in the actual commission of the crime itself, or it may consist of moral assistance to his coconspirators by being present at the commission of the crime or by exerting moral ascendancy over the other co-conspirators. Hence, the mere presence of an accused at the discussion of a conspiracy, even approval of it, without any active participation in the same, is not enough for purposes of conviction. In the instant case, we find petitioners participation in the crime not adequately proven with moral certainty. Undeniably, petitioner, as a private individual, had no hand in the preparation, processing or disbursement of the check issued in her name. A cursory look at the disbursement voucher (No. 101-2002-01-822) reveals the following signatures: signature of Board Member Teodorico Diaz certifying that the cash advance is necessary, lawful and incurred under his direct supervision; signature of Provincial Accountant Camanay certifying to the completeness and propriety of the supporting documents and to the liquidation of previous cash advances; signature of Moises Magallona, Jr. over the name of Provincial Treasurer Cesar M. Cagang certifying that cash is available; signature of Constantino, with the initials of Zoleta adjacent to his name, certifying that the disbursement is approved for payment, and with petitioners signature as the payee. The Sandiganbayan faulted petitioner for immediately encashing the check, insisting that she

should have deposited the check first. Such insistence is unacceptable. It defies logic. The check was issued in petitioners name and, as payee, she had the authority to encash it. All told, there is reasonable doubt as to petitioners guilt. Where there is reasonable doubt, an accused must be acquitted even though his innocence may not have been fully established. When guilt is not proven with moral certainty, exoneration must be granted as a matter of right. PEOPLE OF THE PHILIPPINES VS DIONISIO CALONGE G.R. No. 182793, July 5, 2010 Parricide Facts: Rosita A. Calonge was appellants legitimate wife, with whom he had three children. On December 1, 2001 at around 6:00 oclock in the morning, the Villaverde Police Station received a radio call from the barangay captain of Cabuluan that a massacre took place in their locality. Rositas bloodied body was found lying on the ground about fifteen (15) meters away from their house. Her right hand was loosely clasping a knife. Lying on his back near the stairs was appellant who was also wounded but still conscious. Beside him were a bolo and a flashlight, both stained with blood. While the windows of the house were locked with a piece of tie wire, the door was already opened. Inside the two bedrooms of the house separated only by a curtain, they found the lifeless bodies of the two young girls, Kimberly and Dony Rose. The other child, Melody, was also bloodied but alive and conscious. They brought Melody to the Veterans Regional Hospital where she was treated and confined for seventeen days. Melodys grandparents said they knew it was appellant because they had heard Rosita shouting that appellant will kill them. On the other hand, when appellant was asked what happened and who attacked him, he answered he does not know. Appellant was charged with parricide and

frustrated parricide. Issue: Is the accused guilty of the crime charged? Ruling: YES. Parricide is committed when: (1) a person is killed; (2) the deceased is killed by the accused; (3) the deceased is the father, mother, or child, whether legitimate or illegitimate, or a legitimate other ascendant or other descendant, or the legitimate spouse of accused. The key element in parricide is the relationship of the offender with the victim. All the elements of the crime were clearly and sufficiently proved by the prosecution. Even granting arguendo that Melody did not see the actual stabbing of her mother and two (2) sisters, the attendant circumstances point to no one else but the appellant as the perpetrator. Direct evidence of the actual killing is not indispensable for convicting an accused when circumstantial evidence can sufficiently establish his guilt. The oft-repeated rule has been that circumstantial evidence is adequate for conviction if there is more than one circumstance, the facts from which the inferences are derived have been proven and the combination of all circumstances is such as to produce a conviction beyond reasonable doubt. While no general rule can be laid down as to the quantity of circumstantial evidence which will suffice in a given case, all the circumstances proved must be consistent with each other, consistent with the hypothesis that the accused is guilty, and at the same time inconsistent with the hypothesis that he is innocent, and with every other rational hypothesis except that of guilt. The circumstances proved should constitute an unbroken chain which leads to only one fair and reasonable conclusion that the accused, to the exclusion of all others, is the guilty person. In the killing of victims in this case, the trial court was correct in appreciating the aggravating circumstance of

treachery. There is treachery when the attack is so sudden and unexpected that the victim had no opportunity either to avert the attack or to defend himself. Indeed, nothing can be more sudden and unexpected than when a father stabs to death his two young daughters while they were sound asleep and totally defenseless. PEOPLE OF THE PHILIPPINES VS LUIS ANTONIO GARCHITORENA G.R. No. 184172, May 8, 2009 Parricide Facts: On appeal is the 21 January 2008 Decision of the Court of affirming the conviction of appellant Luis Antonio Garchitorena of the crime of parricide by the Regional Trial Court (RTC) of Quezon City. The accusatory portion of the information reads: That on or about the16th day of [August 2000], in Quezon City, Philippines, the above-named accused, being then the legitimate husband of FLORDELIZA TABLA GARCHITORENA, with intent to kill, did then and there, [willfully], unlawfully and feloniously attack, assault and employ personal violence upon the person of said FLORDELIZA TABL[A] GARCHITORENA, his wife, by then and there shooting her with a gun, hitting her on the head, thereby inflicting upon her serious and mortal wound, which was the direct and immediate cause of her untimely death, to the damage and prejudice of the heirs of said FLORDELIZA TABLA GARCHITORENA. Issue: Is accused guilty of parricide? Ruling: YES.The elements of the crime of parricide are: (1) a person is killed; (2) the deceased is killed by the accused; and (3) the deceased is the father, mother or child, whether legitimate or illegitimate, of the accused or any of his ascendants or descendants, or his spouse.

All the above elements were sufficiently proven by the prosecution. It was stipulated during the pre-trial that appellant and the victim are married on 24 August 1999. That the appellant killed the victim was proven specifically by circumstantial evidence. As aptly stated by the trial court:

In the instant case, the totality of the circumstances warrant a finding that accused is guilty beyond reasonable doubt of the crime charged. The fact that accused and the deceased were the only persons in the bedroom when the shooting incident occurred is undisputed. Secondly, there was an argument between the spouses, as narrated by the accused to the police investigator and during trial. Thirdly, accused, giving no logical excuse, got a gun. In this, the Court finds criminal purpose. Also, there is a finding by this Court of improbability of the deceased shooting herself.

While admittedly there is no direct evidence presented by the prosecution on the killing of the deceased by the accused, the established circumstances aforestated, however, constituted an unbroken chain, consistent with each other and with the hypothesis that the accused is guilty, to the exclusion of all other [hypothesis] that he is not. And when circumstantial evidence constitutes an unbroken chain of natural and rational circumstances corroborating each other, it cannot be overcome by inaccurate and doubtful evidence submitted by the accused.

SULAW 2011
BP 22

VICKY MOSTER VS PEOPLE OF THE PHILPPINES G.R. No. 167461, February 19, 2008

Facts: Petitioner obtained from Presas a loan of P450,000, for which the petitioner issued as payment three postdated PhilBank checks. The three checks were all payable to cash. Presas testified she did not deposit the checks on their due dates upon petitioners request and assurance that they would be replaced with cash. When she could not wait any longer, Presas deposited Check Nos. 026138 and 026124 in her Westmont Bank account, only to be notified later that the checks were dishonored because the account had been closed. Presas said she did not deposit Check No. 026137 after she agreed to petitioners request to withhold its deposit as it had not yet been funded. After receiving notice that Check Nos. 026138 and 026124 had been dishonored, Presas immediately informed petitioner thereof and demanded payment for the value of the checks. This demand, however, went unheeded. In a letter, Presas through counsel, demanded from petitioner the settlement of P367,602, representing the total value of the three checks, within five days from receipt. Petitioner, however, did not comply. Thus, three Informations for violation of B.P. Blg. 22 were filed against petitioner.

Issue: Is petitioner guilty of a violation of BP 22?

Ruling: NO. B.P. Blg. 22 punishes as malum prohibitum the mere issuance of a worthless check, provided the other elements of the offense are proved. Section 1 enumerates the elements of B.P. Blg. 22, as follows: (1) the making, drawing, and issuance of any check to apply on account or for value; (2) the knowledge of the maker, drawer, or issuer that at the time of issue he does not have sufficient funds in or credit with the drawee bank for the payment of the check in full upon its presentment; and (3) the subsequent dishonor of the check by the drawee bank for insufficiency of funds or credit or dishonor for the same reason had not the drawer, without any valid cause, ordered the bank to stop payment. Upon careful examination of the records, however, the Court found that only the first and third elements have been established by the prosecution. By her own admission, petitioner issued the three subject checks, two of which were presented to PhilBank but were dishonored and stamped for the reason Account Closed. Under Section 3 of B.P. Blg. 22, the introduction in evidence of the dishonored check, having the drawees refusal to pay stamped or written thereon, or attached thereto, with the reason therefor as aforesaid shall be prima facie evidence of the making or issuing of the said checks and the due presentment to

the drawee for payment and the dishonor thereof, and that the same was properly dishonored for the reason written, stamped or attached thereto by the drawee on such dishonored checks. As to the second element, Section 2 of B.P. Blg. 22 creates the presumption that the issuer of the check was aware of the insufficiency of funds when he issued a check and the bank dishonored it. This presumption, however, arises only after it is proved that the issuer had received a written notice of dishonor and that, within five days from receipt thereof, he failed to pay the amount of the check or to make arrangements for its payment. Ordinarily, preponderance of evidence is sufficient to prove notice. But in criminal cases, the quantum of proof required is proof beyond reasonable doubt. In the instant case, the prosecution merely presented a copy of the demand letter allegedly sent to petitioner through registered mail and the registry return card. There was no attempt to authenticate or identify the signature on the registry return card. All that we have on record is an illegible signature on the registry receipt as evidence that someone received the letter. As to whether this signature is that of petitioner or her authorized agent remains a mystery. Receipts for registered letters and return receipts do not by themselves prove receipt; they must be properly authenticated to serve as proof of receipt of the letters, claimed to be a notice of dishonor. Unfortunately, the prosecution presented only the testimony of Presas to prove mailing and receipt of the demand letter JAMES SVENDSEN VS PEOPLE OF THE

PHILIPPINES G.R. No. 175381, February 26, 2008 BP 22

Facts: Cristina Reyes (Cristina) extended a loan to petitioner in the amount of P200,000, to bear interest at 10% a month. After petitioner had partially paid his obligation, he failed to settle the balance thereof which had reached P380,000 inclusive of interest. Cristina thus filed a collection suit against petitioner, which was eventually settled when petitioner paid her P200,000 and issued in her favor an International Exchange Bank check postdated February 2, 1999 (the check) in the amount of P160,000 representing interest. The check was co-signed by one Wilhelm Bolton. When the check was presented for payment on February 9, 1999, it was dishonored for having been Drawn Against Insufficient Funds (DAIF). Cristina, through counsel, thus sent a letter to petitioner by registered mail informing him that the check was dishonored by the drawee bank, and demanding that he make it good within five (5) days from receipt thereof. No settlement having been made by petitioner, an Information for violation of BP 22 was filed against

the two.

Issue: Is petitioner guilty of a violation of BP 22?

Ruling: NO. For petitioner to be validly convicted of the crime under B.P. Blg. 22, the following requisites must thus concur: (1) the making, drawing and issuance of any check to apply for account or for value; (2) the knowledge of the maker, drawer, or issuer that at the time of issue he does not have sufficient funds in or credit with the drawee bank for the payment of the check in full upon its presentment; and (3) the subsequent dishonor of the check by the drawee bank for insufficiency of funds or credit or dishonor for the same reason had not the drawer, without any valid cause, ordered the bank to stop payment. Petitioner admits having issued the postdated check to Cristina. The check, however, was dishonored when deposited for payment in Banco de Oro due to DAIF. Hence, the first and the third elements obtain in the case. The evidence for the prosecution failed to prove the second element. While the registry receipt, which is said to cover the letter-notice of dishonor and of demand sent to petitioner, was presented, there is no proof that he or a duly authorized agent received the same. Receipts for registered letters including return receipts do not themselves prove receipt; they must be

properly authenticated to serve as proof of receipt of the letters. Petitioner is civilly liable, however. For in a criminal case, the social injury is sought to be repaired through the imposition of the corresponding penalty, whereas with respect to the personal injury of the victim, it is sought to be compensated through indemnity, which is civil in nature.

ROLANDO L. BALDERAMA VS PEOPLE OF THE PHILIPPINES G.R. Nos. 147578-85, January 28, 2008 Direct Bribery

Facts: Rolando L. Balderama was employed with the Land Transportation Commission (LTO) assigned to the Field Enforcement Division, Law Enforcement Services. Juan S. Armamento, respondent, operates a taxi business with a fleet of ten (10) taxi units. Acting on complaints that taxi drivers in the Ninoy Aquino International Airport discriminate against passengers and would transport them to their destinations only on a contract basis, the LTO created a team to look into the veracity of the complaints. The team flagged down for inspection an SJ Taxi

owned by respondent. The team impounded the taxi on the ground that its meter was defective. However, upon inspection and testing by the LTO Inspection Division, the results showed that contrary to the report of the team, the meter waiting time mechanism of the vehicle was not defective and was functioning normally. The vehicle was released to respondent. Respondent, feeling aggrieved of the malicious impounding of his vehicle, filed with the Office of the Ombudsman a complaint for bribery and violation of Section 3(e) of Republic Act (R.A.) No. 3019, as amended, against herein petitioner. He alleged that prior to the impounding of his taxi, the four LTO officers had been collecting protection money from him.

Issue: Is petitioner guilty of direct bribery?

Ruling: YES. The crime of direct bribery as defined in Article 210 of the Revised Penal Code contains the following elements: (1) that the accused is a public officer; (2) that he received directly or through another some gift or present, offer or promise; (3) that such gift, present or promise has been given in consideration of his commission of some crime, or any act not constituting a crime, or to refrain from doing something which is his official duty to do; and (4) that the crime or act relates to the exercise of his functions as a public officer. The Sandiganbayan found the above elements of direct

bribery present. It was duly established that the accused demanded and received P300.00 as protection money from respondent on several dates. As against the prosecutions evidence, all that the accused could proffer was alibi and denial, the weakest of defenses. To hold a person liable under Section 3(e) of R.A. No. 3019, the concurrence of the following elements must be established beyond reasonable doubt by the prosecution: (1) that the accused is a public officer or a private person charged in conspiracy with the former; (2) that the said public officer commits the prohibited acts during the performance of his or her official duties or in relation to his or her public positions; (3) that he or she causes undue injury to any party, whether the government or a private party; and (4) that the public officer has acted with manifest partiality, evident bad faith or gross inexcusable negligence. The Sandiganbayan found that petitioners and Lubrica participated directly in the malicious apprehension and impounding of the taxi unit of respondent, causing him undue injury. Settled is the rule that findings of fact of the Sandiganbayan in cases before this Court are binding and conclusive in the absence of a showing that they come under the established exceptions, among them: 1) when the conclusion is a finding grounded entirely on speculation, surmises and conjectures; 2) the inference made is manifestly mistaken; 3) there is a grave abuse of discretion; 4) the judgment is based on misapprehension of facts; 5) said findings of facts are conclusions without citation of specific evidence on which they are based; and, 6) the findings of fact of the Sandiganbayan are premised on the absence of evidence

on record. We found none of these exceptions in the present cases.

ZENON R. PEREZ VS PEOPLE OF THE PHILIPPINES G.R. No. 164763, February 12, 2008 Malversation of Public Funds

Facts: An audit team conducted a cash examination on the account of petitioner, who was then the acting municipal treasurer of Tubigon, Bohol. In the course of the audit, the amount of P21,331.79 was found in the safe of petitioner. The audit team embodied their findings in the Report of Cash Examination, which also contained an inventory of cash items. Based on the said audit, petitioner was supposed to have on hand the total amount of P94,116.36, instead of the P21,331.79, incurring a shortage of P72,784.57. When asked by the auditing team as to the location of the missing funds, petitioner verbally explained that part of the money was used to pay for the loan of his late brother, another portion was spent for the food of his family, and the rest for his medicine. As a result of the audit, Arlene R. Mandin prepared a memorandum dated January 13, 1989 addressed to the Provincial Auditor of Bohol

recommending the filing of the appropriate criminal case against petitioner. Petitioner was charged before the Sandiganbayan with malversation of public funds, defined and penalized by Article 217 of the Revised Penal Code

Issue: Is petitioner guilty of malversation?

Ruling: YES. Malversation is defined and penalized under Article 217 of the Revised Penal Code. The acts punished as malversation are: (1) appropriating public funds or property, (2) taking or misappropriating the same, (3) consenting, or through abandonment or negligence, permitting any other person to take such public funds or property, and (4) being otherwise guilty of the misappropriation or malversation of such funds or property. There are four elements that must concur in order that one may be found guilty of the crime. They are: (a) That the offender be a public officer; (b) That he had the custody or control of funds or property by reason of the duties of his office;(c) That those funds or property involved were public funds or property for which he is accountable; and (d) That he has appropriated, took or misappropriated or consented or, through abandonment or negligence, permitted another person to take them.

Evidently, the first three elements are present in the case at bar. At the time of the commission of the crime charged, petitioner was a public officer, being then the acting municipal treasurer of Tubigon, Bohol. By reason of his public office, he was accountable for the public funds under his custody or control. In malversation, all that is necessary to prove is that the defendant received in his possession public funds; that he could not account for them and did not have them in his possession; and that he could not give a reasonable excuse for its disappearance. An accountable public officer may be convicted of malversation even if there is no direct evidence of misappropriation and the only evidence is shortage in his accounts which he has not been able to explain satisfactorily. Verily, an accountable public officer may be found guilty of malversation even if there is no direct evidence of malversation because the law establishes a presumption that mere failure of an accountable officer to produce public funds which have come into his hands on demand by an officer duly authorized to examine his accounts is prima facie case of conversion. Because of the prima facie presumption in Article 217, the burden of evidence is shifted to the accused to adequately explain the location of the funds or property under his custody or control in order to rebut the presumption that he has appropriated or misappropriated for himself the missing funds. Failing to do so, the accused may be convicted under the said provision. However, the presumption is merely prima facie and a rebuttable one. The accountable officer may overcome the presumption by proof to the contrary. If

he adduces evidence showing that, in fact, he has not put said funds or property to personal use, then that presumption is at end and the prima facie case is destroyed. In the case at bar, petitioner was not able to present any credible evidence to rebut the presumption that he malversed the missing funds in his custody or control. RAUL S. TELLO VS PEOPLE OF THE PHILIPPINES G.R. No. 165781, June 5, 2009 Malversation of Public Funds

Facts: Raul S. Tello was a Telegraph Operator and Telegraphic Transfer-in-Charge of the Bureau of Telecommunications. Lordino Tomampos Saligumba, Commission on Audit Auditor II assigned at the office of the Provincial Auditor of Agusan del Sur, received an order directing him and Dionisio Virtudazo to conduct an audit examination of petitioners accounts. Saligumba and Virtudazo conducted an audit where it was initially determined that petitioner had a shortage in the total amount of P6,152.90. When the auditors questioned petitioner on the official receipts of the bank to confirm the remittance advices, petitioner informed them that they were sent to the regional office of the Bureau of Telecommunications. Saligumba wrote the unit auditor of the Philippine National Bank (PNB), San Francisco, Agusan del Sur branch, requesting for confirmation of petitioners remittances and a list of validated remittances from 1 January to 9 December 1986. In a

letter dated 10 December 1986, PNBs branch auditor informed Saligumba that petitioner did not make any remittance to the bank from 31 July 1985 to 30 October 1986. Saligumba secured copies of the official receipts and compared them with the remittance advices submitted by petitioner and found that the banks official receipts did not correspond with petitioners remittance advices. The auditors found that the total shortage incurred by petitioner amounted to P204,607.70. Petitioner failed to submit his explanation and to produce or restitute the missing funds. He was charged before the Sandiganbayan with malversation of public funds under Article 217 of the RPC.

Issue: Is petitioner guilty of malversation?

Ruling: YES. The elements of malversation of public funds under Article 217 of the RPC are: 1. that the offender is a public officer; 2. that he had the custody or control of funds or property by reason of the duties of his office; 3. that those funds or property were public funds or property for which he was accountable; and 4. that he appropriated, took, misappropriated or consented or, through abandonment or negligence, permitted another person to take them. In this case, all the elements of the crime are present. Petitioner is a public officer. He took his Oath of Office as Acting Operator-in-Charge on 13 January

1982. Regional Office Order No. 35 dated 27 September 1984 designated petitioner as Telegraphic Transfer-inCharge aside from his regular duties as Acting Operatorin-Charge of Prosperidad, Agusan del Sur. He was appointed Telegraph Operator effective 1 March 1986. As Telegraph Operator and Telegraphic Transfer-in-Charge, petitioner was in charge of the collections which he was supposed to remit to the PNB. The funds are public funds for which petitioner was accountable. It was also established that petitioner misappropriated the money. He failed to remit his cash collections and falsified the entries in the cashbooks to make it appear that he remitted the money to PNB. Petitioner failed to explain the discrepancies and shortage in his accounts and he failed to restitute the missing amount upon demand. It was also established that petitioner stopped reporting to work starting 8 December 1986. Petitioner did not present any testimonial evidence for his defense. Instead, he merely manifested that he only incurred a shortage of P6,152.90, the initial shortage found by the auditors. The last paragraph of Article 217 of the RPC states: The failure of a public officer to have duly forthcoming any public funds or property with which he is chargeable, upon demand by any duly authorized officer, shall be prima facie evidence that he has put such missing funds or property to personal uses. In this case, petitioner failed to rebut the presumption of malversation. He did not present testimonial evidence to defend himself. He practically admitted the shortage except that he manifested,

contrary to the evidence presented by the prosecution, that only the amount of P6,152.90 was missing. He did not report to his office when the audit examination started. We sustain the Sandiganbayans finding that petitioners guilt has been proven beyond reasonable doubt. VIOLETA BAHILIDAD PHILIPPINES VS PEOPLE OF THE

G.R. No. 185195, March 17, 2010 Malversation of Public Funds Facts: Acting on a complaint filed by a Concerned Citizen of Sarangani Province with the Office of the Ombudsman-Mindanao against Mary Ann Gadian, Amelia Carmela Zoleta, both assigned to the Office of the Vice-Governor, and a certain Sheryll Desiree Tangan, from the Office of the Sangguniang Panlalawigan, for their alleged participation in the scheme of giving fictitious grants and donations using funds of the provincial government, a special audit was conducted in Sarangani province. The Special Audit Team, created for the purpose, conducted its investigation from June 1 to July 31, 2003. Included in the list of alleged fictitious associations that benefited from the financial assistance given to certain NonGovernmental Organizations (NGOs), Peoples Organizations (POs), and Local Governmental Units (LGUs) was Women in Progress (WIP), which received a check in the amount of P20,000.00, issued in the name of herein petitioner Bahilidad, as the Treasurer thereof. Based on its findings, the Special Audit Team recommended the filing of charges of malversation through falsification of public documents against the

officials involved. Issue: Is petitioner guilty of malversation of public funds? Ruling: NO. In the instant case, petitioner was found guilty of conspiring with Zoleta and other public officials in the commission of the crime of Malversation of Public Funds through Falsification of Public Documents. The trial court relied on the dictum that the act of one is the act of all. It is necessary that a conspirator should have performed some overt act as a direct or indirect contribution to the execution of the crime committed. The overt act may consist of active participation in the actual commission of the crime itself, or it may consist of moral assistance to his coconspirators by being present at the commission of the crime or by exerting moral ascendancy over the other co-conspirators. Hence, the mere presence of an accused at the discussion of a conspiracy, even approval of it, without any active participation in the same, is not enough for purposes of conviction. In the instant case, we find petitioners participation in the crime not adequately proven with moral certainty. Undeniably, petitioner, as a private individual, had no hand in the preparation, processing or disbursement of the check issued in her name. A cursory look at the disbursement voucher (No. 101-2002-01-822) reveals the following signatures: signature of Board Member Teodorico Diaz certifying that the cash advance is necessary, lawful and incurred under his direct supervision; signature of Provincial Accountant Camanay certifying to the completeness and propriety of the supporting documents and to the liquidation of previous cash advances; signature of Moises Magallona, Jr. over the name of Provincial Treasurer Cesar M. Cagang certifying that cash is available; signature of

Constantino, with the initials of Zoleta adjacent to his name, certifying that the disbursement is approved for payment, and with petitioners signature as the payee. The Sandiganbayan faulted petitioner for immediately encashing the check, insisting that she should have deposited the check first. Such insistence is unacceptable. It defies logic. The check was issued in petitioners name and, as payee, she had the authority to encash it. All told, there is reasonable doubt as to petitioners guilt. Where there is reasonable doubt, an accused must be acquitted even though his innocence may not have been fully established. When guilt is not proven with moral certainty, exoneration must be granted as a matter of right. PEOPLE OF THE PHILIPPINES VS DIONISIO CALONGE G.R. No. 182793, July 5, 2010 Parricide Facts: Rosita A. Calonge was appellants legitimate wife, with whom he had three children. On December 1, 2001 at around 6:00 oclock in the morning, the Villaverde Police Station received a radio call from the barangay captain of Cabuluan that a massacre took place in their locality. Rositas bloodied body was found lying on the ground about fifteen (15) meters away from their house. Her right hand was loosely clasping a knife. Lying on his back near the stairs was appellant who was also wounded but still conscious. Beside him were a bolo and a flashlight, both stained with blood. While the windows of the house were locked with a piece of tie wire, the door was already opened. Inside the two bedrooms of the house separated only by a curtain, they found the lifeless bodies of the two young girls, Kimberly and Dony Rose. The other child, Melody, was also bloodied but alive and conscious. They brought Melody to the Veterans Regional Hospital where she was treated and

confined for seventeen days. Melodys grandparents said they knew it was appellant because they had heard Rosita shouting that appellant will kill them. On the other hand, when appellant was asked what happened and who attacked him, he answered he does not know. Appellant was charged with parricide and frustrated parricide. Issue: Is the accused guilty of the crime charged? Ruling: YES. Parricide is committed when: (1) a person is killed; (2) the deceased is killed by the accused; (3) the deceased is the father, mother, or child, whether legitimate or illegitimate, or a legitimate other ascendant or other descendant, or the legitimate spouse of accused. The key element in parricide is the relationship of the offender with the victim. All the elements of the crime were clearly and sufficiently proved by the prosecution. Even granting arguendo that Melody did not see the actual stabbing of her mother and two (2) sisters, the attendant circumstances point to no one else but the appellant as the perpetrator. Direct evidence of the actual killing is not indispensable for convicting an accused when circumstantial evidence can sufficiently establish his guilt. The oft-repeated rule has been that circumstantial evidence is adequate for conviction if there is more than one circumstance, the facts from which the inferences are derived have been proven and the combination of all circumstances is such as to produce a conviction beyond reasonable doubt. While no general rule can be laid down as to the quantity of circumstantial evidence which will suffice in a given case, all the circumstances proved must be consistent with each other, consistent with the hypothesis that the accused is guilty, and at the same time inconsistent with the hypothesis that he is innocent, and with every other rational hypothesis

except that of guilt. The circumstances proved should constitute an unbroken chain which leads to only one fair and reasonable conclusion that the accused, to the exclusion of all others, is the guilty person. In the killing of victims in this case, the trial court was correct in appreciating the aggravating circumstance of treachery. There is treachery when the attack is so sudden and unexpected that the victim had no opportunity either to avert the attack or to defend himself. Indeed, nothing can be more sudden and unexpected than when a father stabs to death his two young daughters while they were sound asleep and totally defenseless. PEOPLE OF THE PHILIPPINES VS LUIS ANTONIO GARCHITORENA G.R. No. 184172, May 8, 2009 Parricide Facts: On appeal is the 21 January 2008 Decision of the Court of affirming the conviction of appellant Luis Antonio Garchitorena of the crime of parricide by the Regional Trial Court (RTC) of Quezon City. The accusatory portion of the information reads: That on or about the16th day of [August 2000], in Quezon City, Philippines, the above-named accused, being then the legitimate husband of FLORDELIZA TABLA GARCHITORENA, with intent to kill, did then and there, [willfully], unlawfully and feloniously attack, assault and employ personal violence upon the person of said FLORDELIZA TABL[A] GARCHITORENA, his wife, by then and there shooting her with a gun, hitting her on the head, thereby inflicting upon her serious and mortal wound, which was the direct and immediate cause of her untimely death, to the damage and prejudice of the heirs of said FLORDELIZA TABLA GARCHITORENA.

Issue: Is accused guilty of parricide? Ruling: YES.The elements of the crime of parricide are: (1) a person is killed; (2) the deceased is killed by the accused; and (3) the deceased is the father, mother or child, whether legitimate or illegitimate, of the accused or any of his ascendants or descendants, or his spouse. All the above elements were sufficiently proven by the prosecution. It was stipulated during the pre-trial that appellant and the victim are married on 24 August 1999. That the appellant killed the victim was proven specifically by circumstantial evidence. As aptly stated by the trial court:

In the instant case, the totality of the circumstances warrant a finding that accused is guilty beyond reasonable doubt of the crime charged. The fact that accused and the deceased were the only persons in the bedroom when the shooting incident occurred is undisputed. Secondly, there was an argument between the spouses, as narrated by the accused to the police investigator and during trial. Thirdly, accused, giving no logical excuse, got a gun. In this, the Court finds criminal purpose. Also, there is a finding by this Court of improbability of the deceased shooting herself.

While admittedly there is no direct evidence presented by the prosecution on the killing of the deceased by the accused, the established circumstances aforestated, however, constituted an unbroken chain, consistent with each other and with the hypothesis that

the accused is guilty, to the exclusion of all other [hypothesis] that he is not. And when circumstantial evidence constitutes an unbroken chain of natural and rational circumstances corroborating each other, it cannot be overcome by inaccurate and doubtful evidence submitted by the accused.

PEOPLE OF THE PHILIPPINES VS ESPAOL G.R. No. 175603, February 13, 2009

RENATO

Parricide Facts: At about 2:00 a.m. of February 2, 2000, Domingo Petilla was waiting for his companions at Pantal Road, Dagupan City on their way to Manila. All of a sudden, he heard two successive gunshots. A few moments later, a yellow tricycle sped past him along Pantal Road headed towards Sitio Guibang, Dagupan City. The tricycle was driven by a man wearing a darkcolored long-sleeved shirt. Petillas companions arrived shortly thereafter on board a van. As they started loading their things, they saw, through the lights of their vehicle, a person lying on the pavement along Pantal Road. Upon closer scrutiny, they discovered the lifeless body of Gloria Espaol. The gunshots were also heard by Harold Villanueva, a boatman working at the Pantal River, while he was waiting for passengers at the dock about 100 meters away from the crime scene. The shots were followed by the sound of a motorcycles revving engine. He then saw a speeding yellow tricycle. The tricycle bore the name Rina in front of its cab. Its driver was wearing a dark jacket and blue pants. Out of curiosity, he (the boatman) went there and recognized the victim as one of his regular passengers. Appellant arrived at the scene and

Villanueva noticed that the appellant seemed to be wearing the same clothes as those worn by the driver of the speeding tricycle. He was subsequently charged of parricide. Issue: Is the accused guilty of parricide? Ruling: YES. Under Article 246 of the Revised Penal Code, parricide is the killing of ones legitimate or illegitimate father, mother, child, any ascendant, descendant or spouse and is punishable by the single indivisible penalty of reclusion perpetua to death. None of the prosecution witnesses saw the actual killing of the victim by appellant. However, their separate and detailed accounts of the surrounding circumstances reveal only one conclusion: that it was appellant who killed his wife. Well-entrenched is the rule that the trial courts evaluation of the testimonies of witnesses is accorded great respect in the absence of proof that it was arrived at arbitrarily or that the trial court overlooked material facts. The rationale behind this rule is that the credibility of a witness can best be determined by the trial court since it has the direct opportunity to observe the candor and demeanor of the witnesses at the witness stand and detect if they are telling the truth or not. The Court will not interfere with the trial court's assessment of the credibility of witnesses. In sum, the guilt of appellant was sufficiently established by circumstantial evidence. Reclusion perpetua was correctly imposed considering that there was neither any mitigating nor aggravating circumstance present. The heirs of the victim are entitled to a civil indemnity ex delicto of P50,000, which is mandatory upon proof of the fact of death of the victim and the culpability of the accused for the death. People vs. Tabuelog

G.R. No. 178059, January 22, 2008 Murder

Facts: An Information was filed charging appellant with murder. The accused admits that he stabbed Clinton Badinas on or about that time on said place and as a consequence of the wound he sustained Clinton Badinas died. However, he claimed that he acted in self-defense. The trial court found the version of the prosecution credible thus rejecting appellants theory of self-defense. On May 6, 2005, the trial court rendered a Decision finding appellant guilty of murder. Appellant alleges that the justifying circumstance of selfdefense was not properly considered in his favor; that assuming the killing was committed not in self-defense, still the courts below erred in appreciating the qualifying circumstance of treachery.

Issue: Is appellants contention meritorious?

Ruling: The petition is partly meritorious.

In invoking self-defense, whether complete or incomplete, the onus probandi is shifted to the accused to prove by clear and convincing evidence all the elements of justifying circumstance, namely: (a) unlawful aggression on the part of the victim; (b) the reasonable necessity of the means employed to prevent or repel it; and (c) lack of sufficient provocation on the part of the person defending himself. The accused, in cases of self-defense, must rely on the strength of his own evidence and not on the weakness of the prosecutions evidence since he admits the commission of the alleged criminal act. One who admits the infliction of injuries which caused the death of another has the burden of proving self-defense with sufficient and convincing evidence, for even if the evidence of the prosecution were weak, it could not be disbelieved after the accused himself had admitted the killing. Self-defense, like alibi, is a defense which can easily be concocted. If the accuseds evidence is of doubtful veracity, and it is not clear and convincing, the defense must necessarily fail. We agree with the findings of the trial court as affirmed by the Court of Appeals that the defense miserably In the instant case, appellant failed to establish unlawful aggression on the part of the victim; moreover, his narration of the events was unbelievable. As correctly observed by the trial court, considering the alleged disadvantageous position of the appellant and the relentless assault from the victim, it is surprising that appellant remained unscathed. The presence of a pitcher and a knife conveniently within the reach of appellant was highly suspect and coincidental. As noted by the

trial court, "the presence of a pitcher of water which the accused picked up to repel the attack of the deceased and the knife which the accused was able to grasp and swung it to the (victim) hitting him near the left armpit seems to suggest that pitchers and knives are scattered around Fort Ilocandia."11 Moreover, if it were true that the victim was pursuing Roger Domingo with a broken bottle, then it is preposterous for the appellant to shout at and order Domingo, instead of the victim, to stop, thus putting Domingos life at risk. Further, if Domingo stopped as narrated by appellant, then it is inconceivable that he was not harmed by his alleged pursuer. In fine, the trial court correctly held that the defense failed to prove the element of unlawful aggression on the part of the victim. There being no unlawful aggression, there is no need to discuss whether the means employed to repel the attack was reasonable or whether appellant sufficiently provoked the victim into attacking him. However, we cannot agree with the findings of the trial court that treachery attended the commission of the crime. The trial court appreciated the qualifying circumstance of treachery because "the attack by the accused upon the victim was sudden and coming from behind, thus, precluding any possible way for the victim to defend himself." Nevertheless, mere suddenness of the attack does not amount to treachery. It bears stressing that treachery cannot be presumed. It must be proved with the same quantum of evidence as the crime itself. The fact that the victim might have been unaware or helpless when he was stabbed does not

constitute proof of treachery. The prosecution has the burden to prove that at the time of the attack, the victim was not in a position to defend himself, and that the offender consciously and deliberately adopted the particular means, method and forms of attack employed by him. In the instant case, there was no proof that appellant consciously adopted the mode of attack, hence he may only be held liable for homicide, not murder. The Decision of the Court of Appeals finding appellant guilty of murder is MODIFIED. The Court finds appellant guilty of Homicide. People vs. Gutierrez G.R. No. 188602, February 4, 2010 Murder, Self-defense

Facts: On August 15, 2003, five (5) separate Informations for murder, frustrated murder and three (3) counts of attempted murder were filed against appellant. When arraigned, appellant, with the assistance of counsel de oficio, entered a plea of not guilty to the charges. Trial on the merits then ensued. Not finding credence in appellant's claim of self-defense, the RTC convicted him of murder, frustrated murder and attempted murder on three (3) counts.

Appellant assails the trial court and the CA for giving credence to the prosecutions evidence. He admits having killed Regis and wounding Dalit, but insists that he did so in self-defense.

Issue: Did the accused act in self-defense?

Ruling: No. Self-defense is an affirmative allegation and offers exculpation from liability for crimes only if satisfactorily proved. It requires (a) unlawful aggression on the part of the victim; (b) reasonable necessity of the means employed by the accused to repel it; and (c) lack of sufficient provocation on his part. In People of the Philippines v. Bienvenido Mara, we explained: One who admits killing or fatally injuring another in the name of self-defense bears the burden of proving: (1) unlawful aggression on the part of the victim; (2) reasonable necessity of the means employed to prevent or repel it; and (3) lack of sufficient provocation on the part of the person claiming self-defense. By invoking self-defense, the burden is placed on the accused to prove its elements clearly and convincingly. While all three elements must concur, self-defense relies first and foremost on proof of unlawful aggression on the part of the victim. If no unlawful aggression is proved, no self-

defense may be successfully pleaded. In this case, appellant utterly failed to discharge the burden of proving unlawful aggression. His version of the events was uncorroborated, and his testimony was found to be less credible by the trial court. On the other hand, the surviving victims were unanimous that appellant suddenly fired at them, without any provocation on their part. The credibility of the prosecution witnesses had been weighed by the trial court, and it found their testimonies to be more convincing. As a rule, the appellate court gives full weight and respect to the determination by the trial court of the credibility of witnesses, since the trial judge has the best opportunity to observe their demeanor. While this rule admits of exceptions, none of such exceptions obtains in this case. In Razon v. People, we held: Self-defense cannot be justifiably appreciated when uncorroborated by independent and competent evidence or when it is extremely doubtful by itself. Indeed, in invoking self-defense, the burden of evidence is shifted and the accused claiming self-defense must rely on the strength of his own evidence and not on the weakness of the prosecution. The trial court and the CA cannot, therefore, be faulted for rejecting appellants plea of self-defense. This Court also agrees with the trial court in appreciating treachery as a qualifying circumstance. The essence of treachery is the sudden and unexpected attack by the aggressor on unsuspecting victims, depriving the latter of any real chance to defend themselves, thereby

ensuring its commission without risk to the aggressor, and without the slightest provocation on the part of the victims. The pieces of evidence gleaned by the trial court, the facts, are enough to show that treachery was employed by appellant. The attack was sudden, as testified to by the witnesses, and unexpected. Provocation on the part of the victims was not proven, and appellants testimony that the victims were about to attack him cannot be given credence. The victims had no inkling that an attack was forthcoming and had no opportunity to mount a defense. Thus, treachery was correctly appreciated as a circumstance to qualify the crime to murder. Under Article 248 of the Revised Penal Code (RPC), as amended, the penalty imposed for the crime of murder is reclusion perpetua to death. There being no aggravating or mitigating circumstance, the penalty imposed on appellant is reclusion perpetua, pursuant to Article 63, paragraph 223 of the RPC. The prison term imposed by the trial court in Criminal Case No. 03-3639 is correct. We also affirm the CA ruling that appellant is guilty of attempted murder, not of frustrated murder, in Criminal Case No. 03-3640 for the injury sustained by Dalit. No convincing proof was offered to show that the wound inflicted on Dalit was fatal and would have caused his death had medical help not been provided. It is well settled that where the wounds inflicted on the victim are not sufficient to cause his death, the crime is only attempted murder, as the accused had not performed all the acts of execution that would have brought about the

victim's death. People vs. Dela Cruz G.R. No. 188353, February 16, 2010 Murder qualified by Treachery

Facts: In an Information filed on August 11, 2003, accused-appellant Leozar Dela Cruz y Balobal was indicted for the crime of murder of Vincent Pimentel under Article 248 of the Revised Penal Code. Upon arraignment, he pleaded not guilty to the charge. On September 5, 2006, the RTC rendered its Decision, finding Leozar guilty beyond reasonable doubt of murder attended by treachery and sentencing him to reclusion perpetua. On February 27, 2008, the CA rendered the appealed decision, affirming the findings of the RTC and the conviction of Leozar but modifying the award of damages. Accused raises the same assignment of errors as in his Brief, to wit: first, that the courts a quo erred in appreciating the qualifying aggravating circumstance of treachery; and second, that the courts a quo gravely erred in convicting him of murder instead of homicide.

Issue: Was there treachery?

Ruling: Yes. The fact that Leozar and Vincent did not quarrel prior to the killing is indicative of the treachery employed by Leozar. After Vincent paid Leozar some money, he left and went inside the alley. When Vincent came back to Mockingbird St. from the alley, Leozar deliberately employed means with treachery affording Vincent no opportunity to defend himself, i.e., Leozar draped his arm around Vincent and slash/slit his neck using a 24-inch bladed samurai. The fatal neck wound caused Vincents death, described in his death certificate as "hemorrhagic shock secondary to an incised wound of the neck." All told, the victim was unaware of the imminent attempt on his life, and was not in a position to defend himself. Clearly, treachery was present in this killing. There is treachery when the offender commits any of the crimes against persons, employing means, methods, or forms in the execution, which tend directly and specially to insure its execution, without risk to the offender arising from the defense which the offended party might make. The essence of treachery is that the attack comes without a warning and in a swift, deliberate, and unexpected manner, affording the hapless, unarmed, and unsuspecting victim no chance to resist or escape. For treachery to be considered, two elements must concur: (1) the employment of means of execution that gives the persons attacked no opportunity to defend themselves or retaliate; and (2) the means of execution were deliberately or consciously adopted.

People vs. Tabarnero G.R. No. 168169, February 24, 2010 Self-defense, Voluntary Surrender

Facts: This is an appeal from the Decision of the Court of Appeals in CA-G.R. CR.-H.C. No. 00027 dated April 29, 2005. In said Decision, the Court of Appeals affirmed with modification the August 29, 2002 Decision of the Regional Trial Court (RTC), Branch 78 of Malolos, Bulacan, in Crim. Case No. 888-M-2000, convicting herein appellants Alberto Tabarnero (Alberto) and Gary Tabarnero (Gary) of the crime of Murder. Apellants contended that the court a quo gravely erred in not considering the justifying circumstance of selfdefense and the mitigating circumstance of voluntary surrender interposed by accused-appellant Gary Tabarnero.

Issue: Are the contentions meritorious?

Ruling:

No. The justifying circumstance of self-defense on the part of Gary cannot be considered The requisites for self-defense are: 1) unlawful aggression on the part of the victim; 2) lack of sufficient provocation on the part of the accused; and 3) employment of reasonable means to prevent and repel aggression. The defense invokes the said justifying circumstance, claiming that all of the above three elements are present in the case at bar. There was allegedly unlawful aggression on the part of Ernesto when the latter delivered the first blow with the lead pipe. According to the defense, the means Gary used to defend himself was reasonable, and the shouting shouted professions of his feelings for about Mary Jane could not be considered provocation sufficient for Ernesto to make the unlawful aggression. Unlawful aggression is an indispensable requirement of self-defense of self-defense. As ruled by the Court of Appeals, the evidence presented by Gary to prove the alleged unlawful aggression, namely, his own testimony, is insufficient and self-serving. The alleged sudden appearance of Ernesto and his first attack with the lead pipe the very moment Gary decided to leave seems to this Court to be all too convenient, considering that there was no one around to witness the start of the fight. It also bears to emphasize that by invoking self-defense, Gary, in effect, admitted killing Ernesto, thus, shifting upon him the burden of evidence to prove the elements of the said justifying circumstance. A plea of self-

defense cannot be justifiably appreciated where it is not only uncorroborated by independent and competent evidence, but also extremely doubtful in itself. The defense further argues that assuming that Gary is not qualified to avail of the justifying circumstance of self-defense, he would nevertheless be entitled to the mitigating circumstance of incomplete self-defense under Article 13(1) of the Revised Penal Code. Gary is not entitled to the mitigating circumstance of voluntary surrender The first assignment of error presents another issue for the consideration of this Court. The defense argues that Garys yielding to Alarma should be credited as a mitigating circumstance of voluntary surrender. The Solicitor General agreed with the defense on this point. The Court of Appeals, however, disagreed, and held that the delay of six months before surrendering negates spontaneity, a requisite for voluntary surrender to be considered mitigating. We agree with the Court of Appeals. In order that the mitigating circumstance of voluntary surrender may be credited to the accused, the following requisites should be present: (a) the offender has not actually been arrested; (b) the offender surrendered himself to a person in authority; and (c) the surrender must be voluntary. A surrender, to be voluntary, must be spontaneous, i.e., there must be an intent to submit oneself to authorities, either because he acknowledges his guilt or because he wishes to save them the trouble and expenses in capturing him.

In the case at bar, appellant surrendered to the authorities after more than one year had lapsed since the incident and in order to disclaim responsibility for the killing of the victim. This neither shows repentance or acknowledgment of the crime nor intention to save the government the trouble and expense necessarily incurred in his search and capture. Besides, at the time of his surrender, there was a pending warrant of arrest against him. Hence, he should not be credited with the mitigating circumstance of voluntary surrender. Edgar Esqueda vs. People of the Philippines GR 170222, June 18, 2009 Defense of Alibi

Facts: Edgar Esqueda and one John Doe were charged with two (2) counts of Frustrated Murder in two (2) separate Amended Informations. Accused Edgar entered a plea of not guilty. Accused John Doe remains at-large. On December 12, 2001, the Regional Trial Court (RTC) of Dumaguete City, Branch 33, rendered a Decision acquitting the petitioner in Criminal Case No. 14612 and convicting him in Criminal Case No. 14609. The CA rendered a Decision dated August 19, 2004 dismissing the appeal and affirming the decision of the RTC. Hence, this petition assigning the following error: WHETHER THE TRIAL COURT GRAVELY ERRED IN

FINDING THE PETITONER GUILTY BEYOND REASONABLE DOUBT OF FRUSTRATED HOMICIDE AND IN TOTALLY DISREGARDING HIS DEFENSE. Petitioner's defense is anchored on alibi and denial. His witnesses, Claudio, Domingo and Viviana, aver that during the time of the incident, petitioner was out at sea fishing. Petitioner, when called to the witness stand, denied having committed the crime.

Issue: Should the petition be granted?

Ruling: No. We have unfailingly held that alibi and denial being inherently weak cannot prevail over the positive identification of the accused as the perpetrator of the crime. In the present case, petitioner was positively identified by Venancia and Gaudencio as the author of the crime. Basic is the rule that for alibi to prosper, the accused must prove that he was somewhere else when the crime was committed and that it was physically impossible for him to have been at the scene of the crime. Physical impossibility refers to the distance between the place where the accused was when the crime transpired and the place where it was committed, as well as the facility of access between the two places. Where there is least chance for the accused to be present at the crime scene,

the defense of alibi must fail. Aside from the testimonies of petitioner's witnesses that he was fishing at Cawitan, Sta. Catalina from 8 o'clock in the evening of March 3, 1999 until 2 o'clock in the morning the following day, petitioner was unable to show that it was physically impossible for him to be at the scene of the crime. During the trial of the case, both the prosecution and defense witnesses testified that Nagbinlod and Cawitan, Sta. Catalina, were merely more than 5 kilometers apart which would only take about 20 to 40 minutes ride. Thus, it was not physically impossible for the petitioner to be at the locus criminis at the time of the incident. In addition, positive identification destroys the defense of alibi and renders it impotent, especially where such identification is credible and categorical. People vs. Bucayo G.R. No. 178770, June 13, 2008 Murder qualified by the use of superior strength

Facts: Jonathan Perez and childhood friend Edison Buencillo, Jr. were on their way to visit Jonathans common-law wife, Princess, who lived in Tondo. As they were walking along A. Rivera St., they passed by the group of Fernando and Hector Bucayo and Cesar and Jayson Ortiz, all of whom Jonathan recognized. The

group asked Jonathan and Edison to join them but the two declined the invitation and proceeded to Princess house where they stayed for 15 minutes. They took the same route home and on their way, the group, joined by a certain Pamboy, Fortune, and some others, surrounded and blocked them. As the group taunted and shouted invectives at Jonathan and Edison, a rumble ensued. Jonathan attempted to flee but was dragged back to the melee by Hector. Jonathan saw Hector and Jayson gang up on Edison, as Fernando struck Jonathan repeatedly with a steel chair. As Jonathan was trying to escape, he got hold of a barbecue stick and stabbed Hector with it. Jonathan said he witnessed the assault on Edison and threw stones at the group to make them stop but his attempts were futile. Neither was his call for help heard. He asked for police assistance, and ran to Edisons house to inform the latters mother of the melee. Edison expired at the Jose Reyes Memorial Medical Hospital. After trial, on March 16, 2005, the RTC rendered its decision finding the accused Fernando and Hector guilty beyond reasonable doubt of the crime of murder qualified by superior strength for the death of Edison. The case against Cesar and Jayson were archived to be revived upon their arrest. On April 30, 2007, the CA affirmed with modification the decision of the RTC. The lone issue presented before this Court is: THE TRIAL COURT ERRED IN FINDING THE ACCUSEDAPPELLANTS GUILTY BEYOND REASONABLE DOUBT OF THE CRIME CHARGED DESPITE THE PATENT WEAKNESS OF THE PROSECUTIONS EVIDENCE.

Accused-appellants asserted that the testimony of Jonathan was not credible because he could not have witnessed everything that was happening because he was himself under attack.

Issue: Is accused-appellants assertion meritorious?

Ruling: No. Essentially, accused-appellants claim, that the testimony of Jonathan ought not to be believed simply because Jonathan could not have witnessed the mauling of Edison since he himself was under attack, has no basis. Both the trial and appellate courts found Jonathans testimony credible and their findings should be given full faith and credit. Time and again, we said that the findings by the RTC should be respected as the trial court judge was in the best position to determine the witness credibility. It is well-settled in our jurisdiction that the determination of credibility of witnesses is properly within the domain of the trial court as it is in the best position to observe their demeanor. This conclusion becomes all the more pressing when the appellate court affirms the findings of the trial court. It also bears remembering that people react differently in different situations and there is no standard human response when one is confronted with a strange and frightful experience. Even if a witness is himself attacked, he is still in a position to later on describe

what has transpired. In some situations, when under siege, ones power of observation becomes even more acute and heightened. Recall that at that time Edison was being mauled to death with a steel chair, Jonathan was not himself under siege and even testified that at that time, he was even hurling stones at Edisons maulers. Lastly, the CA found that Jonathan had no reason to fabricate what he witnessed. As against Jonathans straightforward and convincing testimony, the alibi of Fernando that he was asleep in his house and the denial of Hector that they confronted and assaulted Jonathan and Edison miserably fail. Alibi is the weakest of all defenses and as against positive identification by prosecution witnesses, alibi is worthless. Just as alibi is an inherently weak defense, so is denial since these are self-serving negative evidence that cannot be accorded much evidentiary weight than the positive declaration of a credible witness. WHEREFORE, the instant appeal of accused-appellants Fernando and Hector Bucayo is DISMISSED. The April 30, 2007 Decision of the CA is AFFIRMED. People of the Phil. Vs. Zaldy Garcia Y Ancheta G.R. No.174479, June 17, 2008 Crime of murder qualified by treachery

Facts: The prosecution charged the accused-appellant Zaldy

Garcia y Ancheta for the murder of Major Opina qualified by treachery, attended by the special aggravating circumstance of use of an unlicensed firearm, under two separate informations. The charge for violating R.A. 6425 is no longer under review after the RTC acquitted the appellant on ground of reasonable doubt. On arraignment, the appellant pleaded not guilty to the charges laid. On August 26, 2003, the RTC convicted the accused of the crime of murder. The case was elevated to this Court on automatic appeal but was remanded to the Court of Appeals (CA) in accordance with People v. Mateo. The CA decision of May 31, 2006 affirmed with modification the RTC decision. In his brief, the appellant argues that the RTC erred 1. in convicting him after the prosecution failed to prove his guilt beyond reasonable doubt; 2. in appreciating treachery; the qualifying circumstance of

3. in failing to recognize the mitigating circumstance of voluntary surrender in imposing the penalty.

Issue: Should the appeal be granted?

Ruling:

No. Sufficiency of Prosecution Evidence The appellant contends, as his first point, that his guilt has not been proven beyond reasonable doubt; no one really testified that it was he who shot Major Opina. We clarify at the outset that proof beyond reasonable doubt is not solely established by direct evidence. In the absence of direct evidence, the prosecution may present circumstantial evidence that, under given conditions, may meet the evidentiary standard of "proof beyond reasonable doubt" in criminal cases. Circumstantial evidence is sufficient for conviction if: 1) there is more than one circumstance; 2) the facts from which the inferences are derived are proven; and 3) the combination of all the circumstances is such as to produce a conviction beyond reasonable doubt. The conclusions that can be drawn from the chain of proven circumstances rather than their number are material to prove the guilt of the accused. What is paramount is that facts be proven from which inferences may be drawn - with all the circumstances being consistent with one other - that the accused is guilty and this inference is consistent with no other conclusion except that of guilt. The records of this case show that evidence of who actually shot Major Opina is not lacking. In fact, the evidence is the strongest there is, as the appellant himself admitted in open court that he was the one who wielded the gun and pulled the trigger.

The Presence of Treachery There is treachery when the offender commits any of the crimes against persons, employing means, method or forms which tend directly and especially to ensure its execution, without risk to the offender, arising from the defense that the offended party might make. To constitute treachery, two conditions must concur: (1) the employment of means, methods or manner of execution that would ensure the offenders safety from any defense or retaliatory act on the part of the offended party; and (2) the offenders deliberate or conscious choice of the means, method or manner of execution. The appellant seeks to negate these elements of treachery by claiming to have acted out of fear and nervousness; he was allegedly under these stresses because persons who were armed, dressed in civilian clothes and who did not identify themselves as members of the police, scaled his fence. He simply reacted to the intrusion and had no plan to shoot one of those who so approached his house. Hence, he concludes that there was no treachery and the killing could not have been attended by this qualifying circumstance. He posits that the court a quo should have recognized all these. What are the undisputed facts? First, it is not disputed that the appellant went out of his house to see for himself the two men who came. Second, by his own testimony, he returned to his house to get his gun. Third, no immediate shooting took place. The two policemen still called for backup assistance, waited and conferred on what to do, and only after the

backup came did they scale the fence. Twenty minutes must have elapsed from the time the appellant went inside the house up to the time of the actual shooting. Fourth, Major Opina was almost at the door of the appellants house when the shot that killed him rang out. Fifth, the shot came from inside the house through a closed chicken wire screen door that effectively hid a man from inside the house from someone from the outside. Sixth, the first and fatal shot was sudden, immediately hitting Major Opina. We conclude from all these established facts that indeed treachery had attended the killing of Major Opina. While the original initiative originated from the police who sought to arrest the appellant, the latters response was an attack which showed, by its method and manner, that it did not come at the spur of the moment. The appellant was duly forewarned about the identities of Major Opina and SPO4 Oria. Not only was he forewarned, he had ample time to reflect on what to do. His immediate response was to arm himself and to lie in wait in ambush, literally - and to fire from a position of concealment and relative safety at the two policemen who were fully exposed and in the open at the time. The shooting distance of a little more than a meter effectively gave Major Opina no chance. This, in our view, is a classic example of treachery under the definition of the Revised Penal Code of the term.

Voluntary Surrender The essence of voluntary surrender is spontaneity and the intent of the accused to give himself up and submit himself unconditionally to the authorities either because

he acknowledges his guilt or he wishes to save the authorities the trouble and expense that may be incurred for his search and capture. Without these reasons and where the clear reasons for the supposed surrender is the inevitability of arrest and the need to ensure his safety, the surrender cannot be spontaneous and cannot be the "voluntary surrender" that serves as a mitigating circumstance. Again, to hark back to the undisputed facts, no surrender immediately took place after the shooting of Major Opina; what followed was an exchange of shots between the appellant and SPO4 Oria, after which the appellant holed out in his kitchen for some two to three hours. It was only after negotiations with Chief Inspector Lusad that he gave himself up. Thus, SPO3 Benavidez testified that the negotiation was "quite long." SPO4 Oria, on the other hand, testified that the appellant even made demands before he surrendered. When he did surrender, the police had been in place for some time, fully surrounding his house so that he could not have escaped without a major and direct confrontation with them. Then, too, he did not acknowledge liability for the killing of Major Opina even after his surrender to Chief Inspector Lusad. Under these circumstances, none of the attendant elements that would make the surrender a mitigating circumstance was present. The appellant surrendered simply because there was no other way out without risking his own life and limb in a battle with the police. People of the Philippines Vs. Agustino Tamolon, et al. G.R. No. 180169, February 27, 2009

Multiple Murder

Facts: Appellants, with several others, were charged with Multiple Murder, docketed as Criminal Case No. XXI377 (93), before the RTC, Branch 21, Bansalan, Davao del Sur, in an Information which reads:

That sometime last March of 1984, in the Municipality of Magsaysay, Davao del Sur, Philippines and within the jurisdiction of this Honorable Court, the above-named accused, armed with guns and bolos, with intent to kill, and taking advantage of superior strength conspiring, confederating and mutually helping one another, did, then and there willfully, unlawfully and feloniously attack, assault, shoot, hack and massacre Jaime Malabarbas, Ely Malabarbas, Judith Malabarbas, Wilfredo Panton and Gerry Panton, the herein victims/offended parties[,] which gunshot and hack wounds caused to their instantaneous death, to the damage and prejudice of the offended parties. CONTRARY TO LAW.

Upon arraignment, the appellants pleaded not guilty. The RTC rendered its Decision convicting both appellants of multiple murder. The CA rendered its Decision affirming the RTCs with Modification.

Thus, this appeal, assigning the following error: THE COURT A QUO GRAVELY ERRED IN FINDING THE ACCUSED-APPELLANTS AGUSTINO TAMOLON AND ANTONIO CABAGAN GUILTY BEYOND REASONABLE DOUBT OF THE CRIME OF MULTIPLE MURDER ON THE LONE, FABRICATED, ILL-MOTIVATED, AND POLLUTED TESTIMONY OF MODESTO LANDAS.

Issue: Is the appeal meritorious?

Ruling: No. The appellants cast aspersion on the credibility of lone prosecution witness, Modesto Landas, who admitted having been with the armed group that massacred the Malabarbas family. Moreover, they question the motive of Landas who, they said, told the authorities of the alleged criminal activities of the group only after he had been arrested and detained, nine years after the alleged incident. They then submit that "the evidence presented by the prosecution came from a polluted source," harping on Landas being with the roving team at the time of the commission of the crime, making him a co-conspirator. However, the trial court gave full weight and credence to Landas testimony. Evaluating the same, the court said:

Witness Modesto Landas was likewise very positive, direct, straight-forward and convincing in his testimony against accused Agustino Tamolon and Antonio Cabagan. This witness never faltered or wavered in his claim about the participation of accused Agustino Tamolon and Antonio Cabagan in the massacre of the Malabarbas family and in setting fire to the dr[y]er of Vilma Ganad. The CA also held that, by way of exception, the testimony of a co-conspirator may, even if uncorroborated, be sufficient for conviction when it is shown to be sincere in itself, because it is given unhesitatingly and in a straightforward manner, and is full of details by which their nature could not have been the result of a deliberate afterthought. In this regard, worthy of reiteration is the doctrine that on matters involving the credibility of witnesses, the trial court is in the best position to assess the credibility of witnesses, since it has observed firsthand their demeanor, conduct and attitude under grueling examination. Absent any showing of a fact or circumstance of weight and influence which would appear to have been overlooked and, if considered, could affect the outcome of the case, the factual findings on and assessment of the credibility of a witness made by the trial court remain binding on an appellate tribunal. A trial courts assessment of the credibility of a witness is entitled to great weight, even conclusive and binding, if not tainted with arbitrariness or oversight of some fact or circumstance of weight and influence. As to the appellants defense which is based mainly on denial and alibi, nothing is more settled in criminal law jurisprudence than that denial and alibi cannot prevail

over the positive and categorical testimony of the witness. People vs. Domingo G.R. No. 184343, March 2, 2009 Murder and Frustrated Murder, Exemption Criminal Liability in View of Insanity of

Facts: Appellant Jesus Domingo assails the Decision of the Court of Appeals dated 30 April 2008 in CA-G.R. CR No. 30511, modifying the Decision dated 13 November 2006 of Branch 13 of the Regional Trial Court (RTC) of Malolos, Bulacan. The Court of Appeals found appellant guilty beyond reasonable doubt of murder in Criminal Cases No. 1496-M-2000 and No. 1497-M-2000, attempted murder in Criminal Cases No. 1498-M-2000 and No. 1501-M-2000, frustrated murder in Criminal Case No. 1500-M-2000, and frustrated homicide in Criminal Case No. 1499-M-2000. The testimony of the principal witness of the prosecution, Raquel Indon, is assailed by appellant for not being credible due to an inconsistency in her testimony and a lack of conformity with the experience of ordinary men. Appellant also asserts that he was insane or completely deprived of intelligence during the commission of the alleged crimes, and therefore should be exempted from criminal liability in accordance with Article 12, Chapter 2

of the Revised Penal Code.

Issue: Are appellants contention meritorious?

Ruling: No. First contention. Appellant refers to Raquels testimony during cross-examination wherein she narrated that after the appellant entered her bedroom, she screamed. Her sister-in-law, who lived next door, responded by asking Raquel who her assailant was, and the latter identified the appellant. Appellant claims that the conversation between Raquel and her sister-in-law was contrary to the ordinary course of things, and that the initial reaction of people in such a situation would be to ask for help from other people in order to save those who are in danger. Secondly, Raquel also testified during cross-examination that the appellant stabbed the front of her legs when she fell down. It is also argued that the appellant could not have stabbed the front of her legs, since she would be lying on front of her legs when she fell down. This Court finds no merit in these arguments. To begin with, there was nothing out of the ordinary as regards Raquels testimony on these two matters. First, there was nothing unusual about the sister-in-laws query as to who was attacking Raquel. Considering that the

exchange merely consisted of this question and the reply to it, it would not even be accurate to refer to it as a "conversation." Secondly, it was not impossible for the appellant to stab the front of Raquels legs, had her legs been positioned sideways when she fell. But more importantly, these are peripheral details that do not affect the substantial aspects of the incident. Raquel clearly and positively testified that she was carrying her son Marvin when she rushed to the gate and fell down, and the appellant stabbed her legs and thereafter proceeded to stab Marvin who later died from the stab wounds. Her testimony was supported by the MedicoLegal Reports marked as Exhibits "E" and "F." Any inconsistencies in such peripheral details would not exculpate the appellant. Second contention. His claim is not supported by evidence. Appellant offers his uncorroborated testimony as the only proof that he was insane at the time he committed the crime. He testified that nine days before he committed the crime, he suffered from lack of appetite, sleeplessness, and anxiety. In addition, he allegedly heard voices ordering him to kill bad people. He claims that he does not remember anything that happened on 29 March 2000, when the crimes were committed, and that he was already detained when he became conscious of his surroundings. The law presumes every man to be of sound mind. Otherwise stated, the law presumes that all acts are voluntary, and that it is improper to presume that acts are done unconsciously. Thus, a person accused of a crime who pleads the exempting circumstance of insanity has the burden of proving beyond reasonable doubt that he or she was insane immediately before or

at the moment the crime was committed. Insanity exists when there is a complete deprivation of intelligence while committing the act; i.e., when the accused is deprived of reason, he acts without the least discernment because there is a complete absence of power to discern, or there is total deprivation of freedom of the will. Mere abnormality of the mental faculties is not enough, especially if the offender has not lost consciousness of his acts. Insanity is evinced by a deranged and perverted condition of the mental faculties and is manifested in language and conduct. An insane person has no full and clear understanding of the nature and consequences of his or her acts. Even assuming that appellants testimony is credible, his sleeplessness, lack of appetite, nervousness and his hearing imaginary voices, while suggestive of an abnormal mental condition, cannot be equated with a total deprivation of will or an absence of the power to discern. Mere abnormality of mental faculties will not exclude imputability. The popular conception of the word "crazy" is used to describe a person or an act unnatural or out of ordinary. Testimony that a person acted in a crazy or deranged manner days before the commission of the crime does not conclusively prove that he is legally insane and will not grant him or her absolution.
PEOPLE OF THE PHILIPPINES VS. ANSELMO BERONDO JR. G.R. No. 177827, March 30, 2009 Murder to Homicide

Facts:

At around 11:30 p.m. of February 13, 1999, after joining the Miss Gay competition at New Danao, Sinaysayan, Kitaotao, Bukidnon, Herbert Nietes, Jr. walked home to Puntian, Quezon, Bukidnon. While on the way, he suddenly heard a gunshot from nearby. Feeling afraid, he ran towards the grassy area by the roadside to hide. After about five minutes, he saw BERONDO, Julie Tubigon, and Jesus Sudario, each holding a knife, walk towards the road and take turns in stabbing a person who was already slumped on the ground. He recognized the three as they are his townmates. Thereafter, he ran away from the area and went to Bato-Bato, Sinaysayan, Kitaotao, Bukidnon, where he spent the night. The next day, he learned that the person stabbed was GENARO LAGUNA. He later testified that he did not reveal what he had witnessed to anyone because he was afraid of getting involved.

Two years after the incident, Nietes and Tero (another witness) admitted to Dolores, Lagunas widow, that they had witnessed the crime.

Trial proceeded only against accused-appellant BERONDO for murder, because the two other accused remained at-large, where he was convicted. The CA affirmed conviction, but ruled that BERONDO was liable only for homicide.

ISSUES: 1. Does the belated reporting of Nietes of what he witnessed defeat his credibility as a witness? 2. Was the CA correct in holding that BERONDO was liable only for

homicide?

HELD: 1. No. Delay in revealing the identity of the perpetrators of a crime does not necessarily impair the credibility of a witness, especially where sufficient explanation is given.No standard form of behavior can be expected from people who had witnessed a strange or frightful experience. Jurisprudence recognizes that witnesses are naturally reluctant to volunteer information about a criminal case or are unwilling to be involved in criminal investigations because of varied reasons. Some fear for their lives and that of their family; while others shy away when those involved in the crime are their relatives or townmates. And where there is delay, it is more important to consider the reason for the delay, which must be sufficient or well-grounded, and not the length of delay.

Despite the delay in reporting the identities of the malefactors, Nietes testified in a categorical, straightforward, and spontaneous manner, and remained consistent even under grueling cross-examination. Such bears the marks of a credible witness.

2. Yes. The Court finds error in the Trial Courts finding that the killing of the deceased was committed with abuse of superior strength, because no evidence was presented to prove that the accused purposely took advantage of their numerical superiority. Absent clear and convincing evidence of any qualifying circumstance, conviction should only be for homicide.
PEOPLE OF THE PHILIPPINES VS. ROMEO SATONERO @ RUBEN G.R. No. 186233, October 2, 2009

Murder; Self-Defense

FACTS:

At around five oclock in the afternoon of December 25, 1997, Leticia and her nephew, Ramon Amigable were in Brgy. La Esperanza, Tulunan waiting for a tricycle ride to a place called Mlang. Leticia had just received a gift from her sister. Accused-appellant Romeo SATONERO, Leticias nephew too, happened to be nearby. Accusedappellant, upon seeing the gift Leticia was holding, inquired where it came from. When told of the source, accused-appellant mocked the gift-giver for giving more to those who have more in life. Accused-appellant then asked Leticia if she knew who he was, followed by a remark that he would throw her into the irrigation ditch.

At that moment, Leticia told Ramon not to mind accused-appellant because he was drunk. When Ramon was about to board the tricycle, accused-appellant followed him, shot him three times with a short-barreled gun, then stabbed him several times. All told, Ramon sustained nine stab wounds on different parts of his body. Ramon died as a result.

On May 16, 2003, the RTC rendered judgment convicting accused-appellant of murder, discrediting the SATONEROs theory of self-defense. This was affirmed

by the CA.

ISSUE: Did the RTC and CA err in not appreciating self-defense?

HELD:

No. The conviction was proper. There was no selfdefense.

The Court finds no cogent reason to overturn the finding of the CA, confirmatory of that of the RTC, that there was no self-defense on the part of accused-appellant in the instant case.

One who admits killing another in the name of selfdefense bears the onus of proving the justifiability of the killing. The accused, therefore, must convincingly prove the following elements of the justifying circumstance of self-defense: (1) unlawful aggression on the part of the victim; (2) reasonable necessity of the means employed to prevent or repel it; and (3) lack of sufficient provocation on the part of the person claiming selfdefense. While all three elements must concur to support a claim of complete self-defenese, self-defense relies first and foremost on a showing of unlawful

aggression on the part of the victim. Absent clear proof of unlawful aggression on the part of the victim, selfdefense may not be successfully pleaded

In the instant case, accused-appellant failed to discharge his burden of proving unlawful aggression. From a perusal of the trial courts decision, the prosecutions testimonial evidence, notably Leticias testimony, had been carefully weighed and was found by the trial court to be more credible and convincing than the bare and self-serving testimony of accused-appellant as to who initiated the fight and what transpired after the initial assault ensued. The testimony of a single eyewitness to a killing, if worthy of credence, is sufficient to support a conviction for homicide or murder, as the case may be.

The allegation of accused-appellant which pictured Ramon as purportedly pulling out a knife and attempting to stab the former came uncorroborated, although several onlookerspotential witnesses allwere at the situs of the crime. And while claiming to have grappled for some time with Ramon for the possession of the knife, accused-appellant managed to stay unscathed, which in itself is incredible.

PEOPLE OF THE PHILIPPINES VS. PABLO LUSABIO, JR. G.R. No. 186119, Oct. 27, 2009

Murder

FACTS: For the death of Edwin Labini on 12 June 2001, an information was filed on 14 September 2001 before Branch 65 of the RTC of Bulan, Sorsogon, charging accused-appellant Pablo Lusabio, Jr., Tomasito de los Santos and one John Doe with Murder. The case was docketed as Criminal Case No. 01-459.

The Information reads: That on or about 9:00 oclock in the evening of June 12, 2001, at Barangay Biton, municipality of Magallanes, province of Sorsogon, Philippines, and within the jurisdiction of this Honorable Court, the above-named accused, conspiring, confederating and mutually helping one another with intent to kill, treachery, evident premeditation, and abuse of superior strength, did then and there, willfully, unlawfully and feloniously attack, assault and stabbed one Edwin Labini, who sustained mortal/fatal injuries that caused his instantaneous death, to the damage and prejudice of his legal heirs.

On 24 September 2001, based on a complaint of accused-appellant Pablo Lusabio, Jr., an information was filed before the same court charging Tomasito de los Santos, alias Guapo, and Ronnie Dig, alias Tabong, with

Attempted Murder. The case was docketed as Criminal Case No. 01-464.

In Criminal Case No. 01-459 (Murder), the prosecution presented four witnesses, namely: Doris Labini, Dr. Irene V. Ella, Jose Labini and Elsie Gocoyo. In Criminal Case No. 01-464 (Attempted Murder), private complainant Pablo Lusabio, Jr., Dr. Antonio Lopezand Ricardo Cabrera took the witness stand.

The RTC convicted Labini for murder. As to Tomasito de los Santos, the trial court ruled that he had no participation whatsoever in the stabbing of Edwin Labini. The decision was questioned before the CA, alleging insufficiency of evidence, and questioning the credibility of the deceaseds wife. The decision was however affirmed by the CA.

ISSUE: Is the conviction for murder proper?

HELD: Yes. Accused-appellant brands Doris Labini as a biased witness, thus unreliable, because she was the wife of Edwin Labini. The fact that she was the wife of the victim did not necessarily make her a partial witness. It is well-settled that mere relationship of a witness to the victim does not impair the witness credibility. On the contrary, a witness relationship to a victim of a crime would even make his or her testimony more credible, as it would be unnatural for a relative who is interested in vindicating the crime, to accuse somebody other than the real culprit.

In the case at bar, Doris Labini positively identified Pablo Lusabio, Jr. as the one who stabbed her husband. Such declaration was corroborated by the testimony of Tomasito de los Santos that it was, indeed, Lusabio who inflicted the stab wounds on Edwin Labini. Doris Labini was eight meters away from her husband when the latter was stabbed by Lusabio. Aside from this, the crime scene was well-lighted, making it easy for her to identify Lusabio as the perpetrator.

Finally, accused-appellant submits that if ever he committed a crime, he merely committed homicide. He maintains that the prosecution failed to prove that he deliberately and consciously adopted a particular mode of attack in order to eliminate the risk to his person from any defense that Edwin Labini might offer.

The lower court was correct in appreciating treachery in the commission of the crime. There is treachery when the following essential elements are present, viz: (a) at the time of the attack, the victim was not in a position to defend himself; and (b) the accused consciously and deliberately adopted the particular means, methods or forms of attack employed by him. It was clearly established that Edwin Labini, while talking to Pablo Lusabio, Jr. face to face, was suddenly stabbed by the latter with a ten-inch bladed weapon for no reason at all. The suddenness of the stabbing and the fact that Edwin Labini was unarmed gave him no opportunity to defend himself. It is likewise apparent that accused-appellant consciously and deliberately adopted his mode of attack, making sure that the victim would have no chance to

defend himself by reason of the surprise attack.

PEOPLE OF THE PHILIPPINES VS. MARLON DELA CRUZ, ET AL. G.R. No. 174658, February 24, 2009

Anti-Carnapping; Robbery with Homicide (Can one absorb the other?)

FACTS: Two Informations, one for violation of Republic Act No. 6539 (the Anti-Carnapping Law), and the other for Robbery with Homicide, were filed against 1) appellant Marlon dela Cruz (DELA CRUZ), together with 2) Adriano Melecio (Melecio), 3) Jessie Reyes (REYES), and 4) Jepoy Obello (Obello) before the Regional Trial Court (RTC) of Dagupan City. Melecio and Obello have remained at large.

From information gathered from bystanders, the police learned that de la Cruz, a notorious thief who had previously been convicted for theft, and an unidentified man were seen riding on a red Yamaha motorcycle on June 4, 2001, that from a surveillance conducted, de la Cruz was not in his Dagupan residence; and that his mother Maria Rosario (Maria) is living in the municipality of San Quintin. The carnapped motorcycle was owned by a certain Juliana Tamin.

De la Cruzs friends Angelica Perez (Angelica) and Anna Datlag (Anna), who were at the time staying at Marias house, were invited for questioning.

Anna further related: On June 6, 2001, she asked de la Cruz who owns the red motorcycle to which he replied that he took it from an old man who was sleeping after he hit the old man with a stone and Melecio stabbed him at the right side of his body, following which they took the money of the old man. Upon the other hand, de la Cruz put up alibi, claiming that he was asleep in his house at Callejon Extension, Dagupan City on the night of January 3, 2001; that on waking up the following day, January 4, 2001, Obello and Melecio arrived and invited him to, as he did join them to San Quintin on board a motorcycle which the two claimed belongs to their uncle; that the group went first to Lupao, Nueva Ecija where they met Anna and Angelica who, on his invitation, joined them in San Quintin where they stayed for a few days.

After trial, Branch 43 of the Dagupan City RTC convicted DELA CRUZ of both charges. It acquitted Reyes. The conviction was affirmed by the CA.

Among others, DELA CRUZ argues that even if the allegation on the loss of some cash were true, the

same should be absorbed in carnapping since carnapping and robbery have the same element of taking with intent to gain.

ISSUE: Is the contention of DELA CRUZ tenable?

HELD:

No. Carnapping refers specifically to the taking of a motor vehicle. It does not cover the taking of cash or personal property which is not a motor vehicle. As the Court of Appeals noted:

x x x Two (2) articles were taken from TEOFILLO, SR., his tricycle and some cash. The taking of the tricycle constitutes a violation of the anti-carnapping law, RA 6539, while the taking of the cash from tEOFILO, SR. by hitting him with a stone and stabbing him in the chest constitutes the crime of robbery with homicide under Article 294 of the Revised Penal Code.

MUPAS & MUAPAS VS. PEOPLE G.R. No. 172834, February 6, 2008

Frustrated Homicide to Slight Physical Injuries

FACTS:

Petitioners JUN and GIL (aka Bajno) MUPAS were found guilty of frustrated homicide in Criminal Case No. 2314 in the Decision dated 22 November 2002 rendered by the Regional Trial Court of Malaoan, La Union.

The prosecution presented three witnesses, namely: Rogelio Murao (Rogelio - victim), Flaviano Murao (Flaviano) and Dr. Arsenio B. Martinez (Dr. Martinez).

Rogelio testified that at around 7:30 in the morning of 18 February 1993, he was walking to school with his companion Eduardo Murao, Jr. when Jun suddenly stopped and stabbed him using a 29-inch Batangas knife. Meantime, Banjo bodily restrained him but luckily Rogelio was able to avoid the blow. Next, Banjo and Jun hurled stones at him and hit him on the leg while Rogelio was running eastward. Rogelio then flagged down a motorized tricycle but the two assailants continued to pursue him. While inside the tricycle, Banjo held Rogelio by his neck and punched him while Jun stabbed him several times. Then, Rogelio alighted from the tricycle and ran home. Afterwards, his father and mother accompanied him to the hospital. There, Dr. Martinez attended to Rogelio, and found that the wounds

may take TWO WEEKS to HEAL.

Prior to the incident, Rogelio recalled that in January of the same year, he had a misunderstanding with Jun where he and the latter hurled invectives at each other. Rogelio suspected that this event gave rise to the subject incident.

Jun and Gil were found guilty as charged and the judgment of conviction was elevated to the Court of Appeals.

Before the Court of Appeals, Jun and Gil argued that the trial court erred in: (1) finding Gil guilty of the crime charged despite the prosecutions failure to prove his guilt beyond reasonable doubt; and (2) finding Jun guilty of the crime of frustrated homicide instead of physical injuries only. The convictions were however affirmed.

ISSUES: Is the affirmation of the conviction for frustrated homicide proper?

HELD:

No. The trial court solely hinged its judgment of conviction on the victim Rogelios lone and uncorroborated testimony. While it is true that the testimony of one witness is sufficient to sustain a conviction if such testimony establishes the guilt of the accused beyond reasonable doubt, the Court rules in this case that the testimony of one witness in this case is not sufficient for this purpose. It appears then that Rogelio had at his disposal many witnesses who could have supported his allegations but curiously and without any explanation, none of these so-called witnesses were presented. It is thus Rogelios word against the attestations of others. Such omission already raises a reasonable doubt as to the guilt of the petitioners.

Assuming that Gil alias Banjo had any participation, there is likewise no evidence that he or Jun had intent to kill Rogelio. Intent to kill is the principal element of homicide or murder, in whatever stage of commission. Such intent must be proved in a clear and evident manner to exclude every possible doubt as to the homicidal intent of the aggressor.

Although it can be fairly assumed that the injuries suffered by Rogelio were sustained during the fistfight, it is not conclusive that the same were inflicted purposely to kill him. For one, if Jun in fact had been carrying a bolo with intent of killing Rogelio, and if indeed Banjo had conspired with Jun, it is no small wonder why the wounds inflicted were more superficial than mortal, more mild than grave.

Taken in its entirety, there is a dearth of medical evidence on record to sustain the claim that petitioners had any intention to kill Rogelio. When such intent is lacking but wounds were inflicted, the crime is not frustrated homicide but physical injuries only and in this case, less serious physical injuries considering the attending physicians opinion that the wounds sustained by Rogelio would take two (2) weeks to heal.

PEOPLE OF THE PHILIPPINES VS. LEODEGARIO BASCUGIN GR 184704, June 30, 2009 Rape with Homicide; constant change in plea (from guilty to not guilty, and so on); When circumstantial evidence is sufficient for conviction

FACTS:

In an information dated June 21, 1999, BASCUGIN was charged with rape with homicide committed as follows: That on or about the 4th day of June, 1999 at about 7:45 oclock in the evening, at Barangay [XXX], Municipality of Balayan, Province of Batangas, Philippines and within the Jurisdiction of this Honorable Court, the above-named accused, armed with a bladed instrument and a hard object, by means of force and intimidation, did then and there willfully, unlawfully and feloniously have carnal knowledge of [AAA], against her

will and consent and by reason or on the occasion of the said rape, accused with intent to kill, willfully, unlawfully and feloniously stabbed and hit the said AAA, thereby inflicting upon the latter multiple stab wounds and other injuries on the different parts of her body, which caused her instantaneous death. With the assistance of his counsel de oficio, BASCUGIN pleaded guilty upon arraignment on August 5, 1999. Since he was facing a charge for a capital offense, the trial court asked him if his plea was voluntarily given and whether he understood the consequences of his plea. The case then proceeded to trial. In the automatic review by the Supreme Court, the Office of the Solicitor General (OSG) and BASCUGIN challenged the proceedings in the trial court, specifically the invalid arraignment of BASCUGIN. They contended that the consultation made by the counsel de oficio was hasty; and BASCUGIN was not sufficiently apprised of the nature of his case and the consequences of his plea. Finding merit in the OSGs stand, the SUPREME COURT remanded the case to the court a quo for appropriate proceedings. Upon re-arraignment, he pleaded not guilty. Trial then ensued. On September 8, 2003, before the prosecution could rest its case, the defense manifested that BASCUGIN wishes to change his plea of "not guilty" to "guilty." The trial court set his re-arraignment to September 29, 2003 to allow him more time to consider his plea. He was then arraigned on September 29, 2003, and he pleaded guilty to the charge. Then On November 12, 2003, BASCUGIN moved to withdraw his plea of guilty. This was granted

by the trial court in an order dated November 17, 2003. He was re-arraigned on December 1, 2003 and he pleaded "not guilty." He was still found guilty and sentenced to death. This was affirmed by the CA. The appellate court concurred with the trial courts finding that there was sufficient circumstantial evidence pointing to him as the culprit. BASCUGIN went to the SC alleging that there was insufficient evidence to hold him guilty BEYOND reasonable doubt as the prosecution relied on circumstantial evidence. ISSUE: Is BASCUGIN guilty beyond reasonable doubt? HELD: Yes. The decisive factor in BASCUGINs conviction was his admission to the crime when he was examined by his lawyer in court. He testified as follows: xxx Q: Did you feel any remorse or resentment to what happened with you and [AAA]? A: Yes, sir. Q: I noticed also, Mr. Witness, that at the course of the proceedings of this case you are always changing your plea of not guilty/to guilty. Why is it so, Mr. Witness? A: Because I am bothered by my conscience and I was always changing my plea but I feel responsible for what

I did, sir. Q: Do you know fully the consequences of your testimony, Mr. Witness? A: Yes, sir. ATTY. CHAVEZ: I have no more questions, Your Honor. xxx BASCUGINs confession was freely, intelligently, and deliberately given. Judicial confession constitutes evidence of a high order. The presumption is that no sane person would deliberately confess to the commission of a crime unless prompted to do so by truth and conscience. Admission of guilt constitutes evidence against the accused pursuant to the following provisions of the Rules of Court. Furthermore, BASCUGINs confession is consistent with the evidence. We agree with the trial and appellate courts finding that the chain of events constitutes circumstantial evidence that is sufficient to support a conviction. From the testimonies of witnesses and the physical evidence gathered, it was established that the victim was last seen with BASCUGIN in his tricycle; his tricycle was seen parked near a waiting shed in the premises of which the victims personal belongings were later found; his pieces of clothing were found positive for human blood that matches the victims; and the medicolegal report states that BASCUGIN had sexual intercourse with the victim. Circumstantial evidence is that evidence which proves a fact or series of facts from which the facts in issue may

be established by inference. According to Rule 133, Section 4 of the Rules, circumstantial evidence is sufficient for conviction if: (1) there is more than one circumstance; (2) the inference is based on proven facts; and (3) the combination of all circumstances produces a conviction beyond reasonable doubt of the guilt of the accused. In the case at bar, the circumstantial pieces of evidence enumerated by the trial court all point to BASCUGIN as the perpetrator beyond reasonable doubt.
THE PEOPLE OF THE PHILIPPINES VS. RUFINO UMANITO G.R. No. 172607, April 16, 2009 Use of DNA Evidence in Rape

FACTS:

In a Supreme Court Resolution dated 26 October 2007, the Court Resolved, for the very first time, to apply the then recently promulgated New Rules on DNA Evidence (DNA Rules)in a case pending before it this case. The SC remanded the case to the RTC for reception of DNA evidence in accordance with the terms of said Resolution, and in light of the fact that the impending exercise would be the first application of the procedure, directed Deputy Court Administrator Reuben Dela Cruz to: (a) monitor the manner in which the court a quo carries out the DNA Rules; and (b) assess and submit periodic reports on the implementation of the DNA Rules in the case to the Court. To recall, the instant case involved a charge of rape. The

accused Rufino Umanito (UMANITO) was found by the Regional Trial Court (RTC) of Bauang, La Union, Branch 67 guilty beyond reasonable doubt of the crime of rape. UMANITO was sentenced to suffer the penalty of reclusion perpetua and ordered to indemnify the private complainant in the sum of P50,000.00. On appeal, the Court of Appeals offered the judgment of the trial court. UMANITO appealed the decision of the appellate court to this court. In its 2007 Resolution, the Court acknowledged "many incongruent assertions of the prosecution and the defense."At the same time, the alleged 1989 rape of the private complainant, AAA, had resulted in her pregnancy and the birth of a child, a girl hereinafter identified as "BBB." In view of that fact, a well as the defense of alibi raised by UMANITO, the Court deemed uncovering of whether or not UMANITO is the father of BBB greatly determinative of the resolution of the appeal. There is a COMPLETE MATCH in all the fifteen (15) loci tested between the alleles of Rufino UMANITO y Millares and [BBB].
NOTE: UMANITO in this case filed an action to withdraw appeal.

HELD:

By filing such motion, UMANITO is deemed to have acceded to the rulings of the RTC and the Court of Appeals finding him guilty of the crime of rape, and

sentencing him to suffer the penalty of reclusion perpetua and the indemnification of the private complainant in the sum of P50,000.00. Given that the results of the Court-ordered DNA testing conforms with the conclusions of the lower courts, and that no cause is presented for us to deviate from the penalties imposed below, the Court sees no reason to deny UMANITOs Motion to Withdraw Appeal. Consequently, the assailed Decision of the Court of Appeals dated 15 February 2006 would otherwise be deemed final if the appeal is not withdrawn.

Note: IN RE: THE WRIT OF HABEAS CORPUS FOR REYNALDO DE VILLA (detained at the New Bilibid Prisons, Muntinlupa City) vs. THE DIRECTOR, NEW BILIBID PRISONS Nov. 17, 2004

It was argued in this case that DNA analysis on paternity shows conclusively that petitioner de Villa is not the father of Leahlyn Mendoza; his conviction for rape, based on the fact that Leahlyn was sired as a result of the alleged rape, cannot stand and must be set aside.

HELD: Even with all of the compelling and persuasive scientific evidence presented by petitioner and his counsel, we are not convinced that Reynaldo de Villa is

entitled to outright acquittal. As correctly pointed out by the Solicitor General, even if it is conclusively proven that Reynaldo de Villa is not the father of Leahlyn Mendoza, his conviction could, in theory, still stand, with Aileen Mendoza's testimony and positive identification as its bases.The Solicitor General reiterates, and correctly so, that the pregnancy of the victim has never been an element of the crime of rape. Therefore, the DNA evidence has failed to conclusively prove to this Court that Reynaldo de Villa should be discharged.

PEOPLE OF THE PHILIPPINES VS. LEONEL PASAOL PALAC G.R. No. 175600, April 23, 2008 Rape; Alibi as weak defense

On or about the 14th day of May 1996 in Pasay City, Metro Manila, Philippines within the jurisdiction of this Honorable Court, the above-named accused LEONEL PASAOL PALAC alias Joy Joy Talac, in conspiracy with his co-accused REY ARGENTILLO and JOJO VILARDE, by means of force and intimidation they employed upon the person of one [AAA], a minor, 15 years of age, feloniously lay with and have carnal knowledge of said complainant [AAA] against the latter's will, while the two other acc[u]sed watched and stood guard while waiting for [their] turn to have sexual intercourse with the aforesaid [AAA], to her damage and prejudice.

The RTC convicted all three. But PALAC appealed to the CA, pleading defense on the basis of 1) alibi and 2) inconsistencies in the statement of the victim, AAA. The CA however affirmed the conviction.

ISSUE:

Does PALACs appeal have merit?

HELD:

No. Appellant cites the inconsistencies allegedly committed by [AAA]. Allegedly, during her direct examination, [AAA] testified that it was at "6:00 p.m.," when VILARDE offered her a drink, while in paragraph 6 of her Affidavit, she stated it was at 9:00 p.m. [AAA] also contradicted herself on direct examination when she stated that she knocked at the door of the photo[shop] which was opened by ARGENTILLO, when in paragraph 7 of her [A]ffidavit, she alleged that it was VILARDE who knocked at the door and it was appellant who opened the same.A circumspect examination of the record shows that when confronted with the foregoing inconsistencies on cross-examination, [AAA] clarified that it was at 6:00 p.m., not 9:00 p.m., when she was offered a drink by VILARDE; and that it was not she but VILARDE who knocked at the door which appellant opened.
In any event, inconsistencies on matters that transpired prior to the actual commission of the crime and have no bearing to the elements of the crime charged are not treated as proof of a feigning witness but as hallmark of an unrehearsed testimony. Such minor inconsistencies even guarantee truthfulness and candor and serve to strengthen rather than destroy [AAA]'s credibility.

As for appellant's alibi, it does not prosper for he failed to prove, with clear and convincing evidence, that he was in a place other than the place of the crime such that it was physically impossible for him to have committed the crime. The photoshop where AAA was raped was only 15 meters away from the Aragon Compound where the house of CCC in which appellant spent the night on the same date of the incidents.

PEOPLE OF THE PHILIPPINES VS. BENJIE RESURRECCION G.R. No. 185389 Rape

FACTS:

On 20 June 2001, BENJIE was charged before the RTC with Rape under Article 266-A of the Revised Penal Code, as amended by Republic Act No. 8353. He allegedly raped AAA, an 11 year-old girl.

Dr. Marlyn Valdez-Agbayani examined AAA and found that the victim had no laceration in her external organ or her hymen. The former also testified that there were no spermatozoa in the victims vagina. Despite these findings, Dr. Valdez-Agbayani clarified that if the hymen of a woman is elastic and so thin, as in AAAs case, laceration may not be present. As to the absence of spermatozoa in the victims vagina, Dr. Valdez-Agbayani said that it was possible that the victim washed her genitalia, especially

since she was examined only after two days following the alleged rape incident.

BENJIE was convicted by the RTC, which was subsequently affirmed by the CA. Upon appeal to the SC, BENJIE points out that 1) the testimony of AAA on how he allegedly raped her was highly improbable, and 2) the negative findings of spermatozoa and laceration must acquit him.

ISSUE:

Is BENJIE guilty of rape?

HELD:

Yes. This Court itself, in its desire to unveil the truth as borne out by the records, has painstakingly pored over the transcripts of stenographic notes of this case, and like the RTC, finds the victims testimony of the incident candid and straightforward, indicative of an untainted and realistic narration of what transpired on that fateful day.

BENJIE tries to discredit the victim's testimony by questioning

the odd position at which the rape was done. While BENJIEs position, i.e., covering AAAs mouth with his left hand and pinning her down with the right hand, may be considered difficult, such does not exclude the possibility that rape can be consummated under said situation. Depraved individuals stop at nothing in order to accomplish their purpose. Perverts are not used to the easy way of satisfying their wicked cravings. It should be noted that the victim was a very young and fragile 11-year-old, who was easy to be subdued by an abuser who was used to manual labor and was already 18 or 19 years old.

In his last-ditch effort to be exculpated, BENJIE calls this Courts attention to the medical findings that no sperm cells were present in the victims vagina just two days following the rape. He intimates that no rape occurred because of the absence of the sperm cells.

This contention is not well-taken. The absence of spermatozoa in the victims genitalia does not negate rape, the slightest penetration even without emission being sufficient to constitute and consummate the offense. The mere touching of the labia of the womans pudendum or lips of the female organ by the male sexual organ consummates the act.

Note: The court laid down these GUIDING PRINCIPLES in Rape:

To ascertain the guilt or innocence of the accused in cases of rape, the courts have been traditionally guided by three settled principles, namely: (a) an accusation for rape is easy to make, difficult to prove and even more difficult to disprove; (b) in view

of the intrinsic nature of the crime, the testimony of the complainant must be scrutinized with utmost caution; and (c) the evidence of the prosecution must stand on its own merits and cannot draw strength from the weakness of the evidence for the defense.

Since the crime of rape is essentially one committed in relative isolation or even secrecy, it is usually only the victim who can testify with regard to the fact of the forced coitus. In its prosecution, therefore, the credibility of the victim is almost always the single and most important issue to deal with. If her testimony meets the test of credibility, the accused can justifiably be convicted on the basis thereof; otherwise, he should be acquitted of the crime.

PEOPLE MATEO

OF

THE

PHILIPPINES

VS.

NORBERTO

G.R. No. 170569, September 30, 2008 Rape

FACTS:

In a Complaint dated November 2, 1995, AAA, assisted by her father, BBB, charged Norberto MATEO (appellant) with rape by means of force and intimidation. The Assistant City Prosecutor certified that it was filed with the prior authority of the City

Prosecutor.

Upon arraignment, appellant, duly assisted by his counsel, pleaded not guilty to the offense charged.

Upon medical examination, Dr. Reyes testified that AAA could have been laid on a rough surface as shown by the multiple linear abrasions found at her back and the anterum medial aspect of her thigh;that she had been sexually penetrated possibly with the use of force and violence; that he noticed that AAA was suffering from some form of mental retardation as she was not responding to his question like a 17-year old girl should, compelling him to refer her to a neuro-psychiatrist for examination; that based on the result forwarded to him, AAA had a mental age of 5 years and 8 months with an IQ of 38.

After trial, the RTC found MATEO guilty of raping a mental retardate. This was affirmed by the CA.

MATEO questioned the affirmation of the CA in the SC, arguing that 1) there was no proof beyond reasonable doubt there was no physical struggle by the victim, and 2) the court a quo erred in finding that the victim AAA was a mental retardate.

ISSUE:

Is MATEO guilty of rape?

HELD:

Yes. Appellant's claim that the records do not show any sign or presence of struggle is irrelevant. Physical resistance is not an essential element of the felony, and need not be established when intimidation is exercised upon the victim and the latter submits herself, against her will, to the rapist's embrace because of fear for her life and personal safety. It is enough that the malefactor intimidated the complainant into submission. Failure to shout or offer tenacious resistance did not make voluntary the complainant's submission to the criminal acts of the accused.Furthermore, not every victim of rape can be expected to act with reason or in conformity with the usual expectations of everyone.

During the trial, the prosecution presented evidence tending to show that AAA was a mental retardate. It is settled that sexual intercourse with a woman who is a mental retardate constitutes statutory rape, which does not require proof that the accused used force or intimidation in having carnal knowledge of the victim for conviction.

In any event, the prosecution presented adequate evidence which showed that the appellant used force and intimidation in committing the crime of rape, and which the RTC relied upon in convicting appellant.

PEOPLE OF THE PHILIPPINES VS. RODRIGO AWID AND MADUM GANIH G.R. No. 185388, June 16, 2010 Kidnapping for ransom

FACTS: On January 9, 2000 only Mrs. Lee was left in the house, accompanied by three housemaids, and the accused Ernesto Andagao, a gardener-houseboy. They all slept in an extension of the main house, which extension had three rooms. Mrs. Lee was in one with her 11 Japanese Spitz puppies. Next to hers was the room where Andagao slept, and then there was the room of the housemaids.

Part of Mrs. Lees night routine was to let her puppies out of her room about midnight so they could take a leak. At the early dawn of January 10, 2000, after opening the door of her room to let her puppies out, Mrs. Lee was surprised to see a stranger, a man, standing a few meters from her door. She immediately went back in and tried to shut her door close but the man succeeded in pushing the door open and pulling her out of the room just as another man appeared.

Someone struck Mrs. Lee with a gun on both shoulders and kicked her on the ribs. When she fell down, she received a kick on her buttocks. Although she cannot recognized the faces of her abductors because she was blindfolded and covered by black cloth, she noticed that they left Zamboanga City. After traveling three to four hours, they arrived in a house which she later knew that it belonged to a certain Suod Hussain. On January 10, 2000, Mrs. Lee met accused Madum Ganih. She was held for 20 days and during that time she communicated her husband with the order of Ganih to prepare a ransom of P15,000,000. Mr. Lee asked the kidnappers to lower the amount since he could only raise an amount of P1,000,000. Calling her family a third time, the kidnappers reduced their demand to P4 million and threatened to cut off Mrs. Lees head unless this was paid. In the evening of May 5, 2000, Ganih told Mrs. Lee that they would release her the next day. At about 4:00 a.m. of May 6, 2000, her abductors brought Mrs. Lee to Arena Blanco in Zamboanga City where Ganih gave her P100.00 for fare and an M203 bullet as memento. She eventually got home. Sometime after, the police arrested some men which in a police line-up, Mrs. Lee later positively identified as her abductors. For his part, Ganih denied the allegations and claimed an alibi that he was in his house at the said incident. On May 21, 2002 the RTC rendered judgment,convicting Ganih of the crime charged and sentencing him to suffer the penalty of death. The RTC, however, acquitted Awid for insufficiency of evidence.

ISSUE: Is accused Ganih, in conspiracy with others, guilty of kidnapping for ransom?

RULING: To prove the crime charged, the prosecution had to


show (a) that the accused was a private person; (b) that he kidnapped or detained or in any manner deprived another of his or her liberty; (c) that the kidnapping or detention was illegal; and (d) that the victim was kidnapped or detained for ransom. All these have been proved in this case. Significantly, Ganih offered nothing but his bare denial and unsubstantiated alibi to counter the overwhelming evidence that the prosecution adduced against him. His other contention is that the police made Mrs. Lee identify him, not in a proper police line-up but in a mere show-up after giving her some improper suggestions. What the Court condemns are prior or contemporaneous improper suggestions that point out the suspect to the witness as the perpetrator to be identified. Besides, granting that the out-ofcourt identification was irregular, Mrs. Lees court testimony clearly shows that she positively identified Ganih independently of the previous identification she made in front of the police station. Mrs. Lee could not have made a mistake in identifying him since she had ample opportunities to study the faces and peculiar body movements of her kidnappers in her almost four months of ordeal with them.Indeed, she was candid and direct in her recollection, narrating events as she saw them take place. Her testimony, including her identification of the appellant, was positive, straightforward, and categorical. The totality of the prosecutions evidence proves beyond reasonable doubt that Ganih and the others with him kidnapped Mrs. Lee for ransom. The crime was punishable by death at the time of its commission but, with the enactment of Republic Act

9346 that prohibits the imposition of such penalty, the CA was correct in lowering the penalty to reclusion perpetua without eligibility for parole under the Indeterminate Sentence Law.

PEOPLE OF THE PHILIPPINES VS. PABLO ESTACIO AND MARITESS ANG G.R. No. 171655, July 22, 2009 Kidnapping with murder

FACTS: At around 10:00 in the evening of October 10, 1995, Maritess, together with Estacio and Sumipo, arrived at Casa Leonisa, a bar-restaurant at Examiner Street, Quezon City where the three of them would meet with Charlie Mancilla Chua (the victim). Maritess had earlier told Sumipo that she would settle her debt to the victim and then "deretsong dukot na rin x x x kay Charlie [the victim]." Sumipo assumed, however, that Maritess was just joking. Not long after, Estacio pulled out a gun and ordered the victim to pull the car over. As the victim complied, Estacio, with a gun pointed at him, pulled him to the backseat as Maritess transferred to the backseat, sat beside the victim, tied the victims hands behind his back, and placed tape on his mouth. While Sumipo tried to dissuade appellants from pursuing their plan, they replied that they would kill the victim so that he would not take revenge. On Estacios instruction, Sumipo drove towards San Jose del Monte, Bulacan and on reaching a secluded place, Estacio ordered Sumipo to stop the car as he did. Maritess and Estacio then brought the victim to a grassy place. Estacio with bloodied hands later resurfaced. After which, they called the victims mother and demanded money from her. The victims mother having agreed to the demand, Maritess and Estacio directed her to place the money in a garbage can near Pizza Hut in Greenhills at 11:30 in

the evening. Estacio and Sumipo later proceeded to Pizza Hut, and as they were seated there, a patrol car passed by, drawing them to leave and part ways. Sumipo soon learned that Maritess and Estacio sold Chuas gun, watch, and necklace from the proceeds of which he was given P7,000. On May 16, 1996, Sumipo surrendered to the National Bureau of Investigation. On May 23, 1996, Estacio surrendered to the police. The police then informed the victims mother that Estacio had admitted having killed her son, and that he offered to accompany them to the crime scene.

ISSUE: Are the accused guilty of kidnapping for ransom?

RULING: In the case at bar, kidnapping was not sufficiently proven. Although appellants bound and gagged Chua and transported him to Bulacan against his will, they did these acts to facilitate his killing, not because they intended to detain or confine him. As soon as they arrived at the locus criminis, appellants wasted no time in killing him. That appellants intention from the beginning was to kill the victim is confirmed by the conversation which Sumipo heard in the car in which Maritess said that a knife would be used to kill him so that it would not create noise.The subsequent demand for ransom was an afterthought which did not qualify appellants prior acts as kidnapping. where the evident purpose of taking the victims was to kill them, and from the acts of the accused it cannot be inferred that the latters purpose was actually to detain or deprive the victims of their liberty, the subsequent killing of the victims constitute the crime of murder, hence the crime of kidnapping does not exist and cannot be considered as a component felony to produce the complex crime of kidnapping with murder. The crime committed was thus plain Murder. The killing was qualified by treachery. The victim was gagged, bound, and taken from Quezon City to an isolated place in Bulacan against his will to

prevent him from defending himself and to facilitate the killing.

PEOPLE OF THE PHILIPPINES VS. DOMINGO REYES, ALVIN ARNALDO AND JOSELITO FLORES G.R. No. 178300, March 17, 2009 Kidnapping for ransom

FACTS: The Yao family owns and operates a poultry farm in Barangay Santo Cristo, San Jose del Monte, Bulacan. On 16 July 1999, at about 11:00 p.m., the Yao family, on board a Mazda MVP van, arrived at their poultry farm in Barangay Sto. Cristo, San Jose del Monte, Bulacan. Yao San alighted from the van to open the gate of the farm, appellant Reyes and a certain Juanito Pataray (Pataray) approached, poked their guns at Yao San, and dragged him inside the van. Appellant Reyes and Pataray also boarded the van. Thereupon, appellants Arnaldo and Flores, with two male companions, all armed with guns, arrived and immediately boarded the van. Appellant Flores took the drivers seat and drove the van. Appellants Reyes and Arnaldo and their cohorts then blindfolded each member of the Yao family inside the van with packaging tape. Appellant Flores and his male companion told Yao San to produce the amount of five million pesos (P5,000,000.00) as ransom in exchange for the release of Chua Ong Ping Sim, Robert, Raymond and Abagatnan. Thereafter, appellant Flores and his male companion left the van and fled; while Yao San, Lenny, Matthew, Charlene and Josephine remained inside the van. Upon sensing that the kidnappers had already left, Yao San drove the van towards the poultry farm and sought the help of relatives. Meanwhile, Chua Ong Ping Sim, Robert, Raymond and Abagatnan were taken on foot by appellants Reyes and Arnaldo, Pataray and one male companion to a safe-house situated in the

mountainous part of San Jose Del Monte, Bulacan where they spent the whole night. On the morning of 19 July 1999, appellants again called Yao San via a cellular phone and threatened to kill Chua Ong Ping Sim and Raymond because of newspaper and radio reports regarding the incident. Yao San clarified to appellants that he did not report the incident to the police and also pleaded with them to spare the life of Chua Ong Ping Sim and Raymond. Appellants then instructed Yao San to appear and bring with him the ransom of P5 million at 3:00 p.m. in the Usan dumpsite, Litex Road, Fairview, Quezon City. Yao San arrived at the designated place of the pay-off at 4:00 p.m., but none of the appellants or their cohorts showed up. Yao San waited for appellants call, but none came. Thus, Yao San left. On 23 July 1999, the corpses of Chua Ong Ping Sim and Raymond were found at the La Mesa Dam, Novaliches, Quezon CitY.Both died of asphyxia by strangulation.On 26 July 1999, appellant Arnaldo surrendered.

ISSUE: Are the appellants guilty of kidnapping?

RULING: After carefully reviewing the evidence on record and applying the foregoing guidelines to this case, we found no cogent reason to overturn the RTCs ruling finding the testimonies of the prosecution witnesses credible. Prosecution witnesses Abagatnan, Robert, and Yao San positively identified appellants and their cohorts as their kidnappers during a police line-up and also during trial. Abagatnan, Robert and Yao San testified in a clear and candid manner during the trial. Their respective testimonies were consistent with one another. They were steadfast in recounting their ordeal despite the grueling cross examination of the defense. Moreover, their testimonies were in harmony with the documentary evidence adduced by the prosecution. The RTC and

the Court of Appeals found their testimonies credible and trustworthy. Both courts also found no ill motive for Abagatnan, Robert and Yao San to testify against appellants. Although the Yao family was blindfolded during the incident, it was, nevertheless, shown that it took appellants and their cohorts about 10 minutes before all members of the Yao family were blindfolded. During this considerable length of time, Abagatnan, Robert and Yao San were able to take a good look at the faces of appellants and their cohorts. In addition, Abagatnan and Robert narrated that their respective blindfolds loosened several times, giving them the opportunity to have a glimpse at the faces of appellants and their cohorts. It is significant to note that Chua Ong Ping Sim and Raymond were brutally killed as a result of the kidnapping. It is difficult to believe that Robert and Yao San would point to appellants and their cohorts as their kidnappers if such were not true. A witness relationship to the victim of a crime makes his testimony more credible as it would be unnatural for a relative interested in vindicating a crime done to their family to accuse somebody other than the real culprit. Relationship with a victim of a crime would deter a witness from indiscriminately implicating anybody in the crime. His natural and usual interest would be to identify the real malefactor and secure his conviction to obtain true justice for the death of a relative. Finally, we observed that the RTC and the Court of Appeals denominated the crime committed by appellants in the present case as the special complex crime of kidnapping for ransom with double homicide since two of the kidnap victims were killed or died during the kidnapping. PEOPLE OF THE PHILIPPINES VS. SALVINO SUMINGWA G.R. No. 183619, October 13, 2009 Acts of Lasciviousness; Rape; Attempted Rape; Unjust Vexation

FACTS: Sometime in 1999, appellant showed his desire to touch the victim. He fondled the victims breast. On the following

month thereafter, appellant removed the garments of the victim and fondle his penis until it ejaculated. Another incident was on August 2000, wherein the appellant grabbed and lie her down and went top of her and then rubbed her penis into her vaginal orifice, and partially inserted his penis into her vagina. The acts of the appellant continued wherein he would successfully rubbed his organ to her genitalia without penetration. This time, the victim confided to her bestfriend. On December 20, 2000, when the victim and her bestfriend were doing their school work, appellant grabbed the victim, pulled her inside the house and kissed her on the lips. The last incident occurred inside the comfort room of their house on May 27, 2001. When the victim entered, appellant pulled down her short pants and panty, unzipped his trousers, brought out his penis, then repeatedly rubbed it on her vagina while they were in a standing position. The victim decided to report the sexual abuses to her grandmother who forthwith brought her to the National Bureau of Investigation where she was examined by the medico-legal officer. It was found during the examination that there were no extragenital physical injuries on the victims body but there were old, healed, and incomplete hymenal lacerations. Appellant denied all the accusations against him and stated an alibi in his defense.

ISSUE: Is the appellant guilty of the abovementioned cases?

RULING: In her direct testimony, the victim stated that appellant removed her short pants and panty, went on top of her and rubbed his penis against her vaginal orifice. She resisted by crossing her legs but her effort was not enough to prevent appellant from pulling her leg and eventually inserting his penis into her vagina. Clearly, there was penetration.

It is noteworthy that appellant pulled victims leg, so that he could insert his penis into her vagina. This adequately shows that appellant employed force in order to accomplish his purpose. Moreover, in rape committed by a father against his own daughter, the formers moral ascendancy and influence over the latter may substitute for actual physical violence and intimidation. The moral and physical dominion of the father is sufficient to cow the victim into submission to his beastly desires, and no further proof need be shown to prove lack of the victims consent to her own defilement. While appellants conviction was primarily based on the prosecutions testimonial evidence, the same was corroborated by physical evidence consisting of the medical findings of the medico-legal officer that there were hymenal lacerations. When a rape victims account is straightforward and candid, and is corroborated by the medical findings of the examining physician, the same is sufficient to support a conviction for rape. Aside from the fact of commission of rape, the prosecution likewise established that appellant is the biological father of the victim and that the latter was then fifteen (15) 42 years old. Thus, the CA aptly convicted him of qualified rape, defined and penalized by Article 266-B of the RPC. In Criminal Case Nos. 1649 and 1654, although appellant was charged with qualified rape allegedly committed on the second week of November 2000 and May 27, 2001, he should be convicted with Acts of Lasciviousness committed against a child under Section 5(b), Article III of R.A. 7610. The testified that in November 2000, while she and appellant were inside the bedroom, he went on top of her and rubbed his penis against her vaginal orifice until he ejaculated. She likewise stated in open court that on May 27, 2001, while inside their comfort room, appellant rubbed his penis against her vagina while they were in a standing position. In both instances, there was no penetration, or even an attempt to insert his penis into her vagina. The aforesaid acts of the appellant are covered by the definitions of "sexual abuse" and "lascivious conduct" under Section 2(g)

and (h) of the Rules and Regulations on the Reporting and Investigation of Child Abuse Cases promulgated to implement the provisions of R.A. 7610. Appellants acts of embracing, dragging and kissing the in front of her friend annoyed the victim. The filing of the case against appellant proved that the victim was disturbed, if not distressed by the acts of appellant. The appellant is guilty of the following: qualified rape, acts of lasciviousness and unjust vexation.

ELMER DIAMANTE AND TANNY BOY STA. TERESA VS. PEOPLE OF THE PHILIPPINES G.R. No. 180992, September 4, 2009 Robbery and carnapping

FACTS: In the afternoon of July 9, 2000, Cadorniga was in his clinic inside his house when the accused entered knocked therein to seek dental check up. He let them in; he went inside and fix his things. As he got out, he noticed there were already five people inside. He went on with his check up when someone grabbed him and announced hold-up. The assailants soon ransacked the clinic for around 15 minutes and left carrying Cadornigas personal effects. Cadorniga thereafter heard his car alarm sound off, putting him on notice that his car, a Daewoo racer, was likewise taken. At about 10:00 to 11:00 p.m. of the following day, Gerardo turned up at the clinic and advised Cadorniga that they had to rush to Pandacan because his car would be sold to a buyer in Cavite. Accompanied by officers of the Manila police, Gerardo led Cadorniga and his brother to the house of Sta. Teresa who

promptly confessed being one of those who had robbed Cadorniga. Sta. Teresa subsequently led them to the house of Loza where the other accused were hiding. The police thus apprehended Sta. Teresa, Diamante, Maricar, and Lintag and brought them to the police station. Some of the stolen items, including the Daewoo racer, were recovered. Lintag admitted his involvement in the robbery but denied participation in the carnapping. Dela Rosa and Diamante, on the other hand, denied participation and proffered alibi. Meanwhile, Sta. Teresa averred that he was merely helping Maricar and her boyfriend moving things from her mothers house to her new apartment.

ISSUE: Are the assailants guilty of the crime charged?

RULING: The trial and appellate courts found that petitioners were among those who committed robbery and carnapping against Cadorniga as shown by the testimonies of the prosecution witnesses which both courts considered to be straightforward, clear, and consistent. The Court finds no cogent reason to rule otherwise. That Cadorniga was tied down to a stool at gun point to facilitate the commission of the crimes speaks unequivocally that petitioners and their cohorts employed violence and intimidation in taking away Cadornigas personal effects and the Daewoo racer without his consent and with intent to gain. This is clear from the testimony of Cadorniga alone which, as reflected earlier, is categorical on all material points. The records being barren of proof of any ill motive on the part of Cadorniga to testify falsely against petitioners, his testimony is entitled to full faith and credit. Well settled is the rule that the testimony of a single, trustworthy, and credible witness is sufficient for conviction. Gerardos testimony should thus not be doubted merely because his participation was limited to bringing his passengers

to their destination. He positively identified petitioners as among those he had brought to the clinic of Cadorniga and who entered the same on the day of the incident. At the very least, this is further proof of petitioners presence at the crime scene when the robbery and carnapping were committed, belying all uncorroborated allegations to the contrary. The assailants are guilty of both simple robbery and carnapping. PEOPLE OF THE PHILIPPINES VS. LEO QUEMEGGEN G. R. No. 178205, July 25, 2009 Robbery with homicide

FACTS: Noel Tabernilla was driving a passenger jeepney when four of the passengers announced hold-up. After taking the things, the hold-uppers then alighted the jeepney. From there, Tabernilla and his passengers went to the nearest police detachment to report the incident. Three policemen accompanied them to the scene of the crime. While there, the policemen chanced upon the robbers riding a pedicab. Socrates Kagalingan, one of the passengers-victims, recognized the perpetrators, since one of them was still wearing the belt bag that was taken from him. The policemen were able to arrest three suspects, including Janito de Luna, but Leo Quemeggen was able to escape. The three suspects were left under the care of a police officer, Emelito Suing, while the other police officers pursued Quemeggen. Taking advantage of the situation, the three suspects ganged up on Suing; de Luna held his hand, while the other suspect known as "Weng-Weng" shot him on the head.The suspects thereafter escaped. Upon the return of the two policemen who unsuccessfully pursued Quemeggen, Suing was brought to the hospital where he eventually died by a gunshot wound in his head. Appellants

Quemeggen and de Luna were eventually arrested through follow-up operations undertaken by the Navotas Police. Appellants interposed the defense of alibi. They maintained that they were elsewhere when the robbery and shooting incident took place. They claimed that they were in their respective houses: Quemeggen was helping his grandmother cut pieces of cloth used in making rugs, while de Luna was sleeping with his wife.

ISSUE: Can the accused be convicted of a complex crime of robbery with homicide?

RULING: Given the circumstances surrounding the instant case, we agree with the CA that appellants cannot be convicted of Robbery with Homicide. Indeed, the killing may occur before, during, or after the robbery. And it is immaterial that death would supervene by mere accident, or that the victim of homicide is other than the victim of robbery, or that two or more persons are killed. However, essential for conviction of robbery with homicide is proof of a direct relation, an intimate connection between the robbery and the killing, whether the latter be prior or subsequent to the former or whether both crimes are committed at the same time. From the testimonies of the prosecution witnesses, we cannot see the connection between the robbery and the homicide. It must be recalled that after taking the passengers personal belongings, appellants (and two other suspects) alighted from the jeepney. At that moment, robbery was consummated. Some of the passengers, however, decided to report the incident to the proper authorities; hence, they went to the nearest police station. There, they narrated what happened. The police eventually decided to go back to the place where the robbery took place. Initially, they saw no one; then finally, Kagalingan saw the suspects on board a pedicab. De Luna and two other suspects were caught and left under the care of Suing. It was

then that Suing was killed. Clearly, the killing was distinct from the robbery. There may be a connection between the two crimes, but surely, there was no "direct connection." Though appellants were charged with Robbery with Homicide, we find Quemeggen guilty of robbery, and de Luna of two separate crimes of robbery and homicide. It is axiomatic that the nature and character of the crime charged are determined not by the designation of the specific crime, but by the facts alleged in the information. Controlling in an information should not be the title of the complaint or the designation of the offense charged or the particular law or part thereof allegedly violated, these being, by and large, mere conclusions of law made by the prosecutor, but the description of the crime charged and the particular facts therein recited.33 There should also be no problem in convicting an accused of two or more crimes erroneously charged in one information or complaint, but later proven to be independent crimes, as if they were made the subject of separate complaints or information.

PEOPLE OF THE PHILIPPINES VICTOR CALISAY AND JOHN DOE G.R. No. 180507, November 20, 2008 Robbery with homicide

VS.

NESTOR

BAJADA,

FACTS: On December 22, 1999, around 11:30 p.m., while 81year old Villamayor was at home with his 24 year-old live-in partner, Anabelle Asaytono, they heard someone call for Villamayor asking for coffee. The caller introduced himself as "Hector," Villamayor's grandson, but Asaytono recognized the voice as Bajada's. As Villamayor opened the door, the caller, "Hector," pushed the door open with the barrel of a two-foot long gun. Asaytono recognized "Hector" as Bajada because of his

average physique, repulsive smell, the black bonnet which he often wore at work, the deep-set eyes, mouth, a lump on his cheek, and the green shirt which was given to him by Villamayor. Asaytono likewise recognized one of the men as Calisay, noting his hair cut, eye bags, and voice. Calisay wore a red handkerchief across his face and carried a 14-inch knife in his right hand. The third unidentified man, John Doe, wore a bonnet and carried a 2 foot long gun with a magazine. Upon entering the house, John Doe said, "There are many people in Calumpang who are angry at you because you are a usurer engaged in 5-6, so give me PhP 100,000 right now." John Doe made Villamayor sit down but when the latter refused, John Doe made him lie face down on the floor and kicked his back several times. Meanwhile, Bajada pointed his gun at Asaytono and demanded for money. Asaytono denied having any money. She was then made to lie face down on the ground and was kicked. John Doe asked from Villamayor the key to the cabinet which was a meter away from the latter. Villamayor brought out a key from his pocket and handed it to Bajada. Asaytono, who was able to stand up, saw the three accused unlock Villamayor's cabinet and took out its contents which consisted of documents and clothes. Accused-appellants also opened the drawer and took jewelry valued at PhP 80,000 and the PhP 20,000 and USD 500 cash. Thereafter, Bajada pushed Asaytono towards Villamayor, laying her head sideways on Villamayor's head. In this position, Asaytono was able to see Calisay repeatedly stab Villamayor on the back. Calisay then stabbed Asaytono on her left breast. Asaytono pretended to be dead as she lied on Villamayor who was still moving. The three men then hurriedly left the house. Asaytono stood up and saw through the three men move towards the rice field. She noticed that Villamayor's dog wagged its tail as it followed the three men, the way it did when accusedappellants would visit Villamayor. Dr. Marilou Cordon, the medico-legal officer, testified that Villamayor's death was caused by hypovolemic shock secondary to stab wounds. She opined that the stab wounds may have been caused by a single bladed knife inflicted by one person. She

added that the stab which pierced the right lung may have caused his instantaneous death due to blood loss.

ISSUE: Are the accused guilty of the crime of robbery with homicide?

RULING: In any case, Asaytono was able to sufficiently identify Bajada as one of the perpetrators to the satisfaction of the trial court. Asaytono's familiarity with Bajada cannot be denied; she has known Bajada and Calisay for more than a year prior to the incident. The two accused were also frequent visitors at the victim's house. Hence, Asaytono was acquainted with Bajada's physical features. The trial court found her testimony to be credible, frank, straightforward, and consistent throughout the trial. We see no reason to disturb this finding since trial courts are in a unique position to observe the demeanor of witnesses. The trial court's findings regarding the witness' credibility are accorded the highest degree of respect. Furthermore, Bajada could not ascribe any plausible ill motive against the witness. His accusation against Asaytono that the latter was interested in inheriting from Villamayor is self-serving and uncorroborated. Even Bajada's own stepson, Calisay, stated that there was no prior misunderstanding between him and Asaytono and that he did not know any reason why Asaytono would accuse them of a crime. The letters allegedly written by an eyewitness who was afraid to testify in trial cannot be given probative value. The letters accused Asaytono as one of the culpritsa defense which was already dismissed by the courts a quo. There was no evidence to support such allegation. The said letters were belatedly submitted, uncorroborated, and cannot be admitted in evidence Bajada's alibi likewise deserves no merit. For alibi to prosper, it must be shown that the accused was somewhere else at the time of the commission of the offense and that it was physically impossible for the accused to be present at the scene

of the crime at the time of its commission. Bajada himself admitted, however, that the travel time from Bayate, Liliw, Laguna to the crime scene is only 15 minutes by jeep. Hence, it was possible for him to be at the crime scene at or around the time the offense was committed.

PEOPLE OF THE PHILIPPINES VS. EDWIN GAYETA G.R. No. 171654, December 17, 2008 Robbery with rape

FACTS: On 24 July 1995, at around 8:00 p.m., spouses Benjamin and Conchita Nicer were drinking tuba when two armed men barged into their house. One of the armed men, later identified as Arnaldo Reano, was wearing a bonnet while the other, identified as appellant, was wearing a hat. The duo announced a hold-up and ordered the spouses to lie down on the floor. Conchita initially refused to lie down until appellant who incidentally had a bayonet in his other hand, poked a gun at her neck. Reano meanwhile kicked and boxed Benjamin until the latter bled and eventually lost consciousness. Appellant then ordered Conchita to hand over their money. Conchita went up to the room to get P2,500.00 and gave it to appellant. When the duo fled, the Nicer couple reported the incident to the barangay officials who immediately sought police assistance. Meanwhile, spouses BBB and AAA were watching television in their living room when two armed men, also later identified as Reano and appellant, entered their house. They likewise ordered the spouses to lie down and asked them to produce their money. BBB asked AAA to get the money from their store, which was located some twenty (20) meters away from their house. Appellant accompanied AAA to the store while Reano stayed with BBB. Upon reaching the store, AAA took P5,000.00 and gave it to appellant. While in the act of getting the money, appellant

inserted one of his hands inside AAA's short pants. Afterwards, appellant ordered her to undress and lie down on the floor. Appellant also removed his pants, lay on top of AAA, and forcibly had sexual intercourse with her. They went back to the house where appellant also forced AAA to hand over several pieces of jewelry. AAA immediately told BBB that appellant had sexually abused her. The duo fled but came back a few minutes later. Upon seeing them, BBB took the bayonet and tried to stab appellant, but it was deflected by a hard object and fell on the floor. BBB then tried to grab appellant's gun and they grappled for its possession. The gun fired, hitting BBB on his shoulder but he managed to successfully take possession of the gun and fired it twice.

ISSUE: Is the guilt of the appellant was established beyond reasonable ground?

RULING: The Court of Appeals correctly dismissed the inconsistencies in prosecution witness' statements for being trivial and for not having the effect of impairing her credibility as a witness. Inconsistencies as to minor details and peripheral or collateral matters do not affect the credibility of witnesses or the probative weight of their testimonies. Such minor inconsistencies may even serve to strengthen their credibility, as they negate any suspicion that their testimonies are fabricated or rehearsed. Appellant also assails AAA's narration of the rape incident and insinuates that she should have fought off her attacker, given the numerous opportunities presented to her, such as failing to use the bayonet or the bottles that were within her reach to fight off the attacker. Suffice it to say that tenacious resistance against rape is not required; neither is a determined or a persistent physical struggle on the part of the victim necessary. Anent appellant's alibi, it is inherently weak and cannot prevail over a positive identification from a witness found credible

by the trial court. Appellant avers that he was doing his rounds as a member of the Voluntary Lakas Brigade in Muntinlupa, which is nine (9) hours away from Oriental Mindoro, making it physically impossible for him to be at the crime scene. He presented the barangay logbook to support his alibi. The OSG correctly countered that this document was neither authenticated nor identified by the persons who supposedly issued them. All told, the guilt of appellant has been established beyond reasonable doubt.

PEOPLE OF THE PHILIPPINES VS. SAMUEL ALARME AND RIZALDY GELLE G.R. No. 175978, February 12, 2009 Robbery with violence against or intimidations against persons

FACTS: Rudy narrated that he slept at the Maricom Detachment Office located in Punta Cabahug, Cadiz City and rode a tricycle bound for Ceres Bus Terminal at around 2:45 a.m. of September 19, 1995 because his service vehicle broke down. As the tricycle passed by the Cadiz City Park, he saw a parked empty tricycle and an old man being stabbed by three (3) persons. Two (2) persons held the victim while the third one stabbed him. Rudy described the person who stabbed the victim to be "white and tall," while the other two (2) who held the victim were "short." The victim was stabbed several times in front and at the back and cried for help as he was being stabbed. The driver of the tricycle he was riding, apparently afraid, increased the vehicles speed as they passed the stabbing scene. When they reached the Ceres Bus Terminal, he (Rudy) immediately boarded a bus bound for Sagay. He returned to Cadiz on September 21, 1995 and told Cesar Ladiona (Cesar), a barangay tanod, that he saw a person being stabbed at the park in the morning of September 19. Cesar

brought him to the Cadiz City Jail where he was asked whether he could recognize the assailants. He identified the person who stabbed the victim from among the prisoners in jail. He testified on cross-examination that the tricycle he was riding was "very near" the scene of the stabbing incident, and that the park was very brightly lit that night. He stated that he did not immediately report the stabbing incident upon arriving at the Ceres Bus Terminal because he was afraid and because the Ceres bus bound for Sagay was already leaving. When he reported the stabbing incident to Cesar on September 21, 1995, Cesar asked him if he could identify the assailants. He replied that he could, but only through their faces. Cesar then brought him to the city jail where the Chief of Police asked him to point out the persons responsible for the stabbing he reported. He recognized two (2) of the assailants from among the many prisoners inside the jail. He recalled that the prisoners were not brought out of their cell when he was asked to identify the assailants. Norman, a tricycle driver residing in Cadiz City, narrated that he brought his passengers to Ester Pharmacy and Villa Consing, respectively, in the early morning of September 19, 1995; afterwards, he went to Cabahug Street and saw Melanie, the wife of a co-driver. Melanie asked him to look for her (Melanies) husband. Melanie boarded his tricycle and requested to be brought to the Ester Pharmacy. On the way there, he saw Loreto Batarilan (Loreto) driving his own tricycle and trailing his; he also saw three (3) persons walking towards the direction of the Emergency Clinic. He identified two of them as Rizaldy and "Stingray" both of whom he had known for a long time. He went back towards the direction of the City Hall after Melanie alighted at the Ester Pharmacy. He saw Loretos parked tricycle as he passed by the City Hall on Cabahug Street; he then saw Loretos body full of blood lying on the street. He also saw Rizaldy, "Stingray," and a certain John Doe, about "two (2) extended arms length" away from the victims body, walking towards the park carrying a belt bag. He recalled that there were no other persons in the park during that time. He went to the police headquarters to report the incident, but the headquarters was closed. He then went to the Ester Pharmacy and requested the security guard to call the police.

ISSUE: Is the appellant guilty of the crime charged?

RULING: Art. 294. Robbery with violence against or intimidation of persons Penalties. - Any person guilty of robbery with the use of violence against or intimidation of any person shall suffer: 1. The penalty of reclusion perpetua to death, when by reason or on occasion of the robbery, the crime of homicide shall have been committed, or when the robbery shall have been accompanied by rape or intentional mutilation or arson. In the case before us, the RTC convicted the appellants of robbery with homicide based on the testimonies of Rudy, Alicia, and Norman. The CA affirmed this finding without any explanation on how the crime came to be the special complex crime of robbery with homicide. To be sure, Rudys testimony clinched the case against the appellants with respect to the victims stabbing and resulting death. The lower courts apparently deduced the intent to rob from the testimonies of Alicia and Norman. Rizaldy Gelle is convicted of separate crimes of robbery and theft. PEDRO CONSULTA VS. PEOPLE OF THE PHILIPPINES G.R. No. 179462, February 12, 2009 Robbery with intimidation of person

FACTS: At about 2:00 oclock in the afternoon of June 7, 1999, private complainant Nelia R. Silvestre, together with Maria Viovicente and Veronica Amar , boarded a tricycle on their way to Pembo, Makati City. Upon reaching Ambel Street, appellant and his brother Edwin Consulta (Edwin) blocked the tricycle and

under their threats, the driver alighted and left. Appellant and Edwin at once shouted invectives at Nelia, saying "Putang ina mong matanda ka, walanghiya ka, kapal ng mukha mo, papatayin ka namin." Appellant added "Putang ina kang matanda ka, wala kang kadala dala, sinabihan na kita na kahit saan kita matiempuhan, papatayin kita." Appellant thereafter grabbed Nelias 18K gold necklace with a crucifix pendant which, according to an "alajera" in the province, was of 18k gold, and which was worth P3,500, kicked the tricycle and left saying "Putang ina kang matanda ka! Kayo mga nurses lang, anong ipinagmamalaki niyo, mga nurses lang kayo. Kami, marami kaming mga abogado. Hindi niyo kami maipapakulong kahit kailan!" Nelia and her companions immediately went to the Pembo barangay hall where they were advised to undergo medical examination. They, however, repaired to the Police Station, Precinct 8 in Comembo, Makati City and reported the incident. They then proceeded to Camp Crame where they were advised to return in a few days when any injuries they suffered were expected to manifest. Nine days after the incident or on June 16, 1999, Nelia submitted a medico-legal report and gave her statement before a police investigator. Appellant averred that he and his family used to rent the ground floor of Nelias house in Pateros. Nelia is his godmother. The adjacent house was occupied by Nelias parents with whom she often quarreled as to whom the rental payments should be remitted. Because of the perception of the parents of Nelia that his family was partial towards her, her parents disliked his family. Nelias father even filed a case for maltreatment against him which was dismissed and, on learning of the maltreatment charge, Nelia ordered him and his family to move out of their house and filed a case against him for grave threats and another for light threats which were dismissed or in which he was acquitted.

ISSUE: Has the prosecution proved that they accused herein is guilty beyond reasonable ground?

RULING: Animus lucrandi or intent to gain is an internal act which can be established through the overt acts of the offender. It may be presumed from the furtive taking of useful property pertaining to another, unless special circumstances reveal a different intent on the part of the perpetrator. The Court finds that under the above-mentioned circumstances surrounding the incidental encounter of the parties, the taking of Nelias necklace does not indicate presence of intent to gain on appellants part. That intent to gain on appellants part is difficult to appreciate gains light given his undenied claim that his relationship with Nelia is rife with ill-feelings, manifested by, among other things, the filing of complaints6 against him by Nelia and her family which were subsequently dismissed or ended in his acquittal. Absent intent to gain on the part of appellant, robbery does not lie against him. He is not necessarily scot-free, however. From the pre-existing sour relations between Nelia and her family on one hand, and appellant and family on the other, and under the circumstances related above attendant to the incidental encounter of the parties, appellants taking of Nelias necklace could not have been animated with animus lucrandi. Appellant is, however, just the same, criminally liable. The Court finds that by appellants employment of threats, intimidation and violence consisting of, inter alia, uttering of invectives, driving away of the tricycle driver, and kicking of the tricycle, Nelia was prevented from proceeding to her destination. Hence, appellant is guilty of grave coercion and not robbery with intimidation to persons.

PEOPLE OF THE PHILIPPINES VS. ANTONIO ORTIZ, ET AL.

G.R. NO. 179944, SEPTEMBER 4, 2009 ROBERRY WITH RAPE

Facts: The accused-appellant Antonio Ortiz, Charito Chavez and Edwin Dasilio were charged with the crime of robbery with multiple rape. Under the facts, the above-named accused, armed with guns, use force and intimidation against the persons of BBB and AAA at their residence and willfully stole and carried away items of the victims. In addition before leaving, with violence, force and intimidation, at gunpoint succeeded in having carnal knowledge with AAA, one after the other, in taking their turns in satisfying their carnal desires, against her will. The accused alleged alibi as their defense that they were at a place other than Brgy. Xxx, at the time the crime was committed. The trial court ruled that the accused were guilty of the crime charged, under Article 294 of the RPC, as amended by R.A. 7659, and considering the aggravating circumstance that it was committed by an armed band, and with ignominy, sentences all of them, to death. On appeal, the appellate court affirmed the decision of the trial court with modification, instead of the penalty of death; the CA reduced the penalty to reclusion perpetua without eligibility for parole. Hence, this appeal.

Issue: Did the prosecution prove beyond reasonable doubt appellants guilt for the crime of Robbery with Rape?

Ruling: Yes. Under Article 294, Robbery with rape is committed when the following elements concur: (1) the taking of personal

property is committed with violence against or intimidation of persons; (2) the property taken belongs to another; (3) the taking is characterized by intent to gain or animus lucrandi; (4) the robbery is accompanied by rape.

The first three elements were proven by the following established facts: the victims categorically identified appellants as the ones who threatened them and took their personal belongings; all appellants held weapons; appellants entered the house of Candido, herded Candido and his son, Dennis, in a corner of their house and tied their hands; BBB heard the cries of Dennis and when he checked where the cries were coming from, appellants intercepted him and tied his hands as well; appellants entered the house of BBB and AAA, and thereafter ransacked the said house taking valuable items. From the foregoing, it is clear that the crime of robbery was committed.

As to the attendant rape, the courts find the testimony of AAA worthy of full faith and credence. First, records show that AAA cried during her direct examination. Such spontaneous emotional outburst strengthens her credibility. The Supreme Court has held that the crying of the victim during her testimony is evidence of the credibility of the rape charge with the verity born out of human nature and experience. Second, although the examination of Dr. Fajardo of AAAs genital area revealed no laceration in her hymen, it is a settled rule that laceration is not an element of the crime of rape. Simply put, the absence of lacerations does not negate rape. Moreover, hymenal lacerations after sexual congress normally occur on women who have had no prior sexual experience. In this case, AAA is a married woman, who has had prior sexual experience. In the case of People v. Llanita, the Supreme Court noted that the strength and dilatability of the hymen are invariable; it may be so elastic as to stretch without laceration during intercourse. Third, the Supreme Court has held, time and time again, that no woman in her right mind would declare to the whole world that she was raped, unless she is telling the truth. Finally, in the absence of evidence

of improper motive on the part of private complainant AAA to falsely testify against appellants, her testimony deserves great weight and credence.

Regarding appellants defense of alibi, the same cannot prevail over the positive identification of appellants as perpetrators of the crime charged. For alibi to prosper, it is not enough for the appellants to prove that they were somewhere else when the crime was committed. They must further demonstrate that it was physically impossible for them to have been at the scene of the crime at the time of its commission. Here, appellants interposed the alibi that they were at a place other than Brgy. xxx, at the time the crime was committed; however, no one corroborated their testimonies.

PEOPLE OF THE PHILIPPINES vs. FO1 FELIPE DELA CRUZ et al G.R. No. 168173, DECEMBER 24, 2008 ROBBERY WITH HOMICIDE, ROBBERY IN BAND

Facts: This is a review on automatic appeal of the decision of the CA that fully affirmed the decisionof the RTC. The RTC decision found the accused-appellants FO1 dela Cruz, Audie, Alfredo, Eduardo, Bernardo, Joemari, Dominador, and Robert guilty of the special complex crime of robbery with homicide and robbery in band. Accordingly, the RTC sentenced them to suffer the death penalty for robbery with homicide, and an indeterminate penalty of six (6) years prision correccional, as

minimum, to ten (10) years of prision mayor, as maximum, for robbery in band.

Issue: Is appellant guilty of the crime charged?

Ruling: I. Criminal Case No. Q-99-85788 (Robbery in Band) In the present case, the prosecution witnesses, at one time or another during the hearing, testified that Joemari, Bernardo, Diosdado and FO1 dela Cruz were all armed. However, we cannot recognize the commission of robbery by a band as an aggravating circumstance since this circumstance was not specifically alleged in the body of the Information. Section 8, Rule 110 of the 2000 Rules on Criminal Procedure provides that the information or complaint must state the designation of the offense given by the statute and specify its qualifying and generic aggravating circumstances Ruben, the driver of the vehicle the appellants used and who saw the robberies from the start to its bloody end, positively and with full details identified in his testimony of September 28, 1999 the appellants Robert, Eduardo, Audie, Bernardo, Dominador, Joemari, Alfredo, and Diosdado as the robbers. At gunpoint, they boarded his jeepney in Monumento; ordered him to refuel at Petron Gas Station in Commonwealth, Quezon City and robbed this establishment; and then ordered him to stop at the 7-Eleven Convenience Store along Mindanao and Tandang Sora Avenue for another robbery. Rommel, in his testimony of October 21, 1999, corroborated the testimony of Ruben and likewise gave his own details of how the robbery was committed. He identified Audie, FO1 dela Cruz, Diosdao and Joemari as the passengers of the jeepney whom he recognized.

These testimonies, which we considered in light of the appellants defenses discussed below, more than amply constitute proof beyond reasonable doubt that the appellants are guilty of the crime of robbery as charged. II. Criminal Homicide) Case No. Q-99-85787 (Robbery with

In the case before us, the prosecution proved that the appellants original intention was to rob the 7-Eleven Convenience Store. A careful examination of the testimonies of the various prosecution witnesses, all of them cited above, reveals the following facts showing the appellants intent: appellants Joemarie, Bernardo and Robert entered the 7-Eleven Convenience Store pretending to be customers; witness Kuraishi entered the store and met appellant Bernardo, who was carrying a gun; Elmer, who went out of his car to follow Kuraishi, was shot in the chest by Diosdado; appellant FO1 dela Cuz fired at the security guard, Nestor, through the glass door but missed; Nestor exchanged shots with FO1 dela Cruz; Joemari pulled down Edwin and took his wallet and watch; Diosdado peeped through the glass panel of the 7-Eleven Convenience Store, shot Nestor and entered the store; Joemari dragged Edwin towards the counter and told him to open the cash register; Diosdado went outside the store, approached the two (2) taxis parked in front of King Dimsum and held up the drivers; FO1 dela Cruz entered the store, dragged the cashier, Rose, towards the backroom and asked who kept the keys of the cash register; Joemarie, Bernardo, Robert and FO1 dela Cruz took the cash register and went back to their companions who were waiting inside the jeepney; thereafter, appellants proceeded to Paniqui, Tarlac. From the foregoing, the overriding intention of the appellants could not but be to rob the 7-Eleven Convenience Store; the killings were merely incidental, resulting by reason or on the occasion of the robbery. Nestor was killed because he was the man who would have resisted the robbery; Elmer was killed because he simply happened to be there as the robbery was taking place. LUIS MARCOS P. LAUREL VS. HON, ZEUS C. ABROGAR etc,

et al. G.R. NO. 155076, JANUARY 13, 2009 THEFT

Facts: Petitioner Luis P. Laurel is charged with the crime of theft under Article 308 of the RPC. Under the facts, petitioner, with intent to gain and without consent from Philippine Long Distance Telephone (PLDT), stole and use the international long distance calls belonging to PLDT by conducting International Simple Resale (ISR), which is a method of routing and completing international long distance calls using lines, cables, antennae, and/or air wave frequency which connect directly to the local or domestic exchange facilities of the country where the call is destined, effectively stealing this business from PLDT. Petitioner filed a motion to quash, on the ground that the factual allegations in the Amended Information do not constitute the felony of theft. The trial court denied the motion. Petitioners special civil action for certiorari was dismissed by the Court of Appeals. Thus, petitioner filed the instant petition for review with this Court.

Issue: Is petitioners act constitute theft of respondent PLDTs business and service?

Ruling: Yes. Under Article 308 of RPC, Theft is committed by any person who, with intent to gain but without violence against, or intimidation of persons nor force upon things, shall take

personal property of another without the latters consent.

Since the passage of the Revised Penal Code on December 8, 1930, the term "personal property" has had a generally accepted definition in civil law. In Article 335 of the Civil Code of Spain, "personal property" is defined as "anything susceptible of appropriation and not included in the foregoing chapter (not real property)." To appropriate means to deprive the lawful owner of the thing. The word "take" in the Revised Penal Code includes any act intended to transfer possession which, as held in the assailed Decision, may be committed through the use of the offenders own hands, as well as any mechanical device, such as an access device or card as in the instant case.

In the instant case, the act of conducting ISR operations by illegally connecting various equipment or apparatus to private respondent PLDTs telephone system, through which petitioner is able to resell or re-route international long distance calls using respondent PLDTs facilities constitutes all three acts of subtraction. The acts of "subtraction" include: (a) tampering with any wire, meter, or other apparatus installed or used for generating, containing, conducting, or measuring electricity, telegraph or telephone service; (b) tapping or otherwise wrongfully deflecting or taking any electric current from such wire, meter, or other apparatus; and (c) using or enjoying the benefits of any device by means of which one may fraudulently obtain any current of electricity or any telegraph or telephone service. The business of providing telecommunication or telephone service is likewise personal property which can be the object of theft under Article 308 of the Revised Penal Code. Business may be appropriated under Section 2 of Act No. 3952 (Bulk Sales Law), hence, could be object of theft:

Therefore, the business of providing telecommunication and the telephone service are personal property under Article

308 of the Revised Penal Code, and the act of engaging in ISR is an act of "subtraction" penalized under said article. However, the Amended Information describes the thing taken as, "international long distance calls," and only later mentions "stealing the business from PLDT" as the manner by which the gain was derived by the accused. In order to correct this inaccuracy of description, this case must be remanded to the trial court and the prosecution directed to amend the Amended Information, to clearly state that the property subject of the theft are the services and business of respondent PLDT. Parenthetically, this amendment is not necessitated by a mistake in charging the proper offense, which would have called for the dismissal of the information under Rule 110, Section 14 and Rule 119, Section 19 of the Revised Rules on Criminal Procedure. To be sure, the crime is properly designated as one of theft. The purpose of the amendment is simply to ensure that the accused is fully and sufficiently appraised of the nature and cause of the charge against him, and thus guaranteed of his rights under the Constitution.

ERNESTO PIDELI vs.PEOPLE OF THE PHILIPPINES G.R. No. 163437, February 13, 2008 THEFT

Facts: Placido and Wilson entered into a verbal partnership agreement to subcontract a rip-rapping and spillway project. Petitioner Pideli, brother to Wilson and neighbor and friend to Placido, offered the duo the use of his credit line with the Mt. Trail Farm Supply and Hardware (MTFSH) in La Trinidad, Benguet. With the said arrangement, Wilson and Placido, with the assistance of petitioner, were able to secure an assortment of

construction materials for the rip-rap and spillway contract. After the completion of the project, Placido, Wilson and petitioner computed their expenses and arrived at a net income of P130,000.00. Placido, as partner, claimed one-half (1/2) or P65,000.00 of the net amount as his share in the project. Petitioner, however, advised the two to first settle their accountabilities for the construction materials taken from the hardware store. Placido and Wilson did as told and entrusted the full amount to petitioner, with express instructions to pay MTFSH and deliver the remaining balance to them. The following day Placido attempted but failed to contact petitioner. He had hoped to obtain his share of the partnership income. Placido got hold of petitioner the next morning. Unexpectedly, petitioner informed Placido that nothing was left of the proceeds after paying off the supplier.Despite repeated demands, petitioner refused to give Placido his share in the net income of the contract.

Alarmed over the sudden turn of events, Placido lodged a complaint for theft against petitioner Ernesto Pideli. The trial court found petitioner guilty of theft and upon appeal, the CA affirmed the conviction.

Issue: Did the trial court erred in finding petitioner guilty of theft?

Ruling:

No. Accordingly, the elements of theft are as follows: That there be taking of personal property; That said property belongs to another; That the taking be done with intent to gain; That the taking be done without the consent of the owner; and That the taking

be accomplished without the use of violence against or intimidation of persons or force upon things. There is, here, a confluence of the elements of theft. Petitioner received the final payment due the partners Placido and Wilson under the pretext of paying off their obligation with the MTFSH. Under the terms of their agreement, petitioner was to account for the remaining balance of the said funds and give each of the partners their respective shares. He, however, failed to give private complainant Placido what was due him under the construction contract.

Although there is misappropriation of funds here, petitioner was correctly found guilty of theft. As early as U.S. v. De Vera, the Court has consistently ruled that not all misappropriation is estafa. Chief Justice Ramon C. Aquino, in his commentary on the Revised Penal Code, succinctly opined: The principal distinction between the two crimes is that in theft the thing is taken while in estafa the accused receives the property and converts it to his own use or benefit. However, there may be theft even if the accused has possession of the property. If he was entrusted only with the material or physical (natural) or de facto possession of the thing, his misappropriation of the same constitutes theft, but if he has the juridical possession of the thing, his conversion of the same constitutes embezzlement or estafa. The record bears out that private complainant originally claimed P65,000.00 as his share in the partnership. However, he admitted receiving the total amount of P15,500.00, on two separate occasions, from Wilson Pideli. Verily, only P49,500.00 is due private complainant.

Hence, the imposable penalty is the maximum period of prision mayor minimum and medium prescribed in the abovequoted first paragraph of Article 309. That period ranges from six (6) years and one (1) day to ten (10) years, plus one (1) year for every additional ten thousand pesos in excess of P22,000.00, which in this case is two (2) years for the excess amount of P27,500.00. Applying the Indeterminate Sentence Law, the maximum term could be twelve (12) years while the minimum term would fall under the next lower penalty of prision correccional in its medium and maximum periods (2 years, 4 months and 1 day to 6 years), to be imposed in any of its periods.

ROSE AOAS VS. PEOPLE OF THE PHILIPPINES G.R. NO. 155339, MARCH 3, 2008 THEFT

Facts: Petitioner Rose Aoas was charged for the crime of theft of eighteen (18) sacks of red and white beans, all valued at P24,720.00 belonging to NATY MADON-EP. During the trial, prosecution presented two witnesses, the private complainant and barangay tanod Gregorio Garcia. In summary, private complainant testified that she was engaged in the business of selling assorted beans, and adjacent of her stall in the market was that of accused appellant. She reported the matter to the authorities that 18 sacks of red and white beans were missing.

Upon inquiry from the persons in the city market she was informed by a certain Gregorio Garcia that the accused-appellant was the culprit. The defense proffered an explanation which, unfortunately, was not given credence. Defense witness Imelda Bautista testified that she was also engaged in the buying and selling of beans. Her goods were also kept at the second floor/mezzanine of petitioner's stall. Petitioner testified that she brought out sacks of beans from her stall because one Ronda Sabado bought them from Imelda Bautista. The trial court held petitioner guilty of the crime of theft. On appeal, the Court of Appeals affirmed the RTC decision in toto.

Issue: Is petitioner guilty of the crime of theft?

Ruling: No. Considering that there is no direct evidence pointing to petitioner as the perpetrator of the crime, the trial court relied solely on circumstantial evidence. Circumstantial evidence is that evidence which proves a fact or series of facts from which the facts in issue may be established by inference. It is founded on experience, observed facts and coincidences establishing a connection between the known and proven facts and the facts sought to be proved. In order that conviction be had, the following must concur: 1. There is more than one circumstance; 2. The facts from which the inferences are derived are proven; 3. The combination of the circumstances is such as to produce a conviction beyond reasonable doubt.

To uphold a conviction based on circumstantial evidence, it is essential that the circumstantial evidence presented must constitute an unbroken chain which leads one to a fair and reasonable conclusion pointing to

the accused, to the exclusion of the others, as the guilty person. The test to determine whether or not the circumstantial evidence on record is sufficient to convict the accused is that the series of circumstances duly proved must be consistent with one other and that each and every circumstance must be consistent with accused's guilt and inconsistent with his innocence. The circumstances must be proved, and not themselves presumed. The circumstantial evidence must exclude the possibility that some other person has committed the offense. After a careful review, the court finds that the aforesaid circumstantial evidence does not pass this test of moral certainty as to warrant petitioner's conviction. Complainant testified that 18 sacks of beans which she stored in the mezzanine of her stall were missing. She discovered the loss in the morning when she saw red and white beans scattered on the floor in front of her stall and that of petitioner.She accused herein petitioner as the culprit after being informed by barangay tanod Gregorio Garcia that he saw petitioner in the evening riding in a jeepney loaded with sacks of beans. Garcia alleged that he was only 30 meters away from the jeepney and the place was sufficiently lighted, enough for him to recognize that the sacks loaded in the jeepney contained beans. The fact that beans were scattered on the floor inside and in front of the stall of petitioner and in the parking lot does not necessarily lead to the conclusion that petitioner is the perpetrator of the crime. The prosecution has failed to show that the circumstances invoked completely discount the possibility that persons other than petitioner could have

perpetrated the crime. Thus, where the proven facts and circumstances are capable of two or more explanations, one of which is consistent with innocence and the other with guilt, the evidence does not fulfill the test of moral certainty and is not sufficient to convict the accused.
PEOPLE OF THE PHILIPPINES VS. REYNALDO BAYON Y RAMOS G.R. NO. 168627, JULY 2, 2010 QUALIFIED THEFT

Facts: Appellant Reynaldo Bayon Y Ramos was charged of the crime of theft for having stolen; 1 Rado Diastar wrist watch, 1 Seiko Divers watch and 1bolo belonging to Eduardo Cunnanan . He was also charged with the crime of qualified theft for being, then a stay-in helper of Arturo Limoso y Loot at his residence and for having stolen valuable items belonging to him. The pre-trial was terminated without stipulations. Thereafter, joint trial of the cases ensued. The trial court held appellant guilty for the crime of qualified theft. However, for the crime of theft, he was acquitted for the crime charged on the ground of reasonable doubt. Appellant appealed to the Court of Appeals, contending that the circumstantial evidence presented against him by the prosecution was insufficient to prove his guilt beyond reasonable doubt, and that there was nothing whatsoever that would link him to the commission of the crime of theft. However, the Court of Appeals affirmed the decision of the trial court. Hence, this appeal.

Issue: Did the Court of Appeals erred in finding appellant guilty

beyond reasonable doubt of the crime of qualified theft?

Ruling: Yes. Under Article 310 of the RPC, heft becomes qualified "if committed by a domestic servant, or with grave abuse of confidence, or if the property stolen is a motor vehicle, mail matter or large cattle, or consists of coconuts taken from the premises of a plantation, fish taken from a fishpond or fishery, or if property is taken on the occasion of fire, earthquake, typhoon, volcanic eruption, or any other calamity, vehicular accident or civil disturbance."

In this case, the Court of Appeals affirmed the trial courts conviction of appellant based on circumstantial evidence. For circumstantial evidence to be sufficient for conviction, the following conditions must be satisfied: (a) There is more than one circumstance; (b) The facts from which the circumstances are derived are proven; and (c) The combination of all the circumstances is such as to produce a conviction beyond reasonable doubt. Circumstantial evidence suffices to convict an accused only if the circumstances proved constitute an unbroken chain which leads to one fair and reasonable conclusion that points to the accused, to the exclusion of all others as the guilty person; the circumstances proved must be consistent with each other, consistent with the hypothesis that the accused is guilty, and at the same time inconsistent with any other hypothesis except that of guilty. The two pieces of circumstantial evidence cited by the trial court and affirmed by the appellate court do not form an unbroken chain that point to appellant as the author of the crime; hence, their conclusion

becomes merely conjectural. Notably, the prosecution failed to establish the element of unlawful taking by appellant. Since appellants statement during the custodial investigation was inadmissible in evidence as he was not assisted by counsel, the prosecution could have presented the person to whom appellant allegedly sold the pieces of jewelry as witness, but it did not do so. It could have been the missing link that would have strengthened the evidence of the prosecution. The Court finds the circumstantial evidence relied upon by the trial and appellate courts in convicting appellant to be insufficient in proving his guilt beyond reasonable doubt absent any substantial evidence of unlawful taking by appellant.

PEOPLE OF THE PHILIPPINES VS. RODOLFO GALLO Y GADOT G.R. NO. 187730, JUNE 29, 2010 ESTAFA

Facts: Accused-appellant Rodolfo Gallo was charged with illegal recruitment and estafa. For the crime of estafa, under the facts the said accused by means of false manifestations and fraudulent representations which they made to the latter, prior to and even simultaneous with the commission of the fraud, to the effect that they had the power and capacity to recruit and employ said EDGARDO V. DELA CAZA in Korea as factory worker and could facilitate the processing of the pertinent papers if given the necessary amount to meet the requirements thereof; induced

and succeeded in inducing said EDGARDO V. DELA CAZA to give and deliver to said accused the amount of P45,000.00 on the strength of said manifestations and representations. Accusedappellant denied having any part in the recruitment of Dela Caza. In fact, he testified that he also applied with MPM Agency for deployment to Korea as a factory worker. The RTC rendered its Decision convicting the accused of syndicated illegal recruitment and estafa. On appeal, the CA held the totality of the prosecutions evidence showed that the accused-appellant, together with others, engaged in the recruitment of Dela Caza. His actions and representations to Dela Caza can hardly be construed as the actions of a mere errand boy. The evidence presented establishes his liability for estafa under paragragh 2(a) of Article 315 of the Revised Penal Code.

Issue: Did the trial court erred in finding accused-appellant guilty of estafa?

Ruling: No. The prosecution likewise established that accusedappellant is guilty of the crime of estafa as defined under Article 315 paragraph 2(a) of the Revised Penal Code, viz: Any person who shall defraud another by any means mentioned herein below 2. By means of any of the following false pretenses or fraudulent acts executed prior to or simultaneously with the commission of the fraud: (a) By using fictitious name, or falsely pretending to possess power, influence, qualifications, property, credit, agency, business or imaginary transactions; or by means of other similar deceits.

The elements of estafa in general are: (1) that the accused defrauded another (a) by abuse of confidence, or (b) by

means of deceit; and (2) that damage or prejudice capable of pecuniary estimation is caused to the offended party or third person.Deceit is the false representation of a matter of fact, whether by words or conduct, by false or misleading allegations, or by concealment of that which should have been disclosed; and which deceives or is intended to deceive another so that he shall act upon it, to his legal injury.

All these elements are present in the instant case: the accused-appellant, together with the other accused at large, deceived the complainants into believing that the agency had the power and capability to send them abroad for employment; that there were available jobs for them in Korea as factory workers; that by reason or on the strength of such assurance, the complainants parted with their money in payment of the placement fees; that after receiving the money, accused-appellant and his co-accused went into hiding by changing their office locations without informing complainants; and that complainants were never deployed abroad. As all these representations of the accused-appellant proved false, paragraph 2(a), Article 315 of the Revised Penal Code is thus applicable.
PEOPLE OF THE PHILIPPINES VS. RODOLFO GALLO G.R. NO. 185277, MARCH 18, 2010 ESTAFA

Facts: Appellant Rodolfo Gallo (Gallo), together with Pilar Manta (Manta) and Fides Pacardo (Pacardo), was originally charged with illegal recruitment in large scale and estafa filed before the Regional Trial Court. The prosecution likewise presented

documentary evidence consisting of the promissory notes and official receipts issued by the agency to the private complainants. Also presented was a certification issued by the Philippine Overseas Employment Agency, stating that according to its records, the New Filipino Manpower Development and Services, Inc. had an expired license and that its application for the reissuance of a new license was denied. For his defense, appellant Gallo alleged that he was not an employee of MPM but was himself an applicant for overseas work. The trial court rendered a decision convicting him of the crimes charged. Accused Manta and Pacardo were acquitted for insufficiency of the evidence presented against them. In view of the penalty imposed, the case was elevated to the Supreme Court for automatic review. However, the Court resolved to transfer the cases to the Court of Appeals for intermediate review. The CA rendered the accusedappellant Rodolfo Gallo shall be credited with the full extent of his preventive imprisonment pursuant to Article 29 of the Revised Penal Code.

Issue: Did the trial court erred in finding the accused-appellant guilty of the crime of estafa?

Ruling: No. The Supreme Court was convinced that the prosecution was able to prove, beyond reasonable doubt, appellants guilt for estafa under Article 315 (2)(a) of the Revised Penal Code, which provides: 1. By means of any of the following false pretenses or fraudulent acts executed prior to or simultaneously with the commission of the fraud: (a)By using fictitious name, or falsely pretending to possess power, influence, qualifications, property, credit, agency, business or imaginary transactions, or by means of other similar deceits.

Under the above-quoted provision, there are three (3) ways of committing estafa: (1) by using a fictitious name; (2) by falsely pretending to possess power, influence, qualifications, property, credit, agency, business or imaginary transactions; and (3) by means of other similar deceits.To convict for this type of crime, it is essential that the false statement or fraudulent representation constitutes the very cause or the only motive which induces the complainant to part with the thing of value.

In the case before us, appellant and Martir led the private complainants to believe that they possessed the power, qualifications and means to provide work in Korea. During the trial of these cases, it was clearly shown that, together with Martir, appellant discussed with private complainants the fact of their being deployed abroad for a job if they pay the processing fee, and that he actually received payments from private complainants. Thus, it was proven beyond reasonable doubt that the three private complainants were deceived into believing that there were jobs waiting for them in a factory in Korea when in fact there were none. Because of the assurances of appellant, each of the private complainants parted with their money and suffered damages as a result of their being unable to leave for Korea. The elements of estafa deceit and damage are thus indisputably present, making the conviction for estafa appropriate.

PEOPLE OF THE PHILIPPINES VS. LOURDES LO, GRACE CALIMON AND AIDA COMILA G.R. NO. 175229, JANUARY 29, 2009 ESTAFA

Facts: Three separate complaints were filed charging Lourdes Lo (Lo) and accused-appellants Grace Calimon (Calimon) and Aida Comila (Comila) with illegal recruitment andestafa. Upon arraignment, herein accused-appellants pleaded "not guilty" to the crimes charged. Accused Lo, however, has not yet been apprehended and has remained at large. Accused-appellant Calimon denied the accusations against her. She claimed that she was also an applicant for overseas job placement and that she never promised any work abroad to private complainants. She averred that it was Lo who recruited her and private complainants. Accused-appellant Comila, on the other hand, denied having known or seen Lo. However, she maintained that it was accused Lo who recruited and received money from private complainants. She averred she could not have recruited private complainants because she gave birth in Baguio. The RTC rendered a Decision convicting the appellants of the crimes charged. On appeal, the CA affirmed the Decision of the RTC but with modifications.

Issue: Did the trial court erred in convicting accused-appellants for the crime of estafa?

Ruling: No. The Supreme Court was likewise convinced that the prosecution proved beyond reasonable doubt that accusedappellants are guilty of estafa under Article 315(2)(a) of the Revised Penal Code, which provides: 2. By means of any of the following false pretenses or fraudulent acts executed prior to or simultaneously with the commission of the fraud: (a) By using

fictitious name, or falsely pretending to possess power, influence, qualifications, property, credit, agency, business or imaginary transactions; or by means of other similar deceits.

There are three ways of committing estafa under the above-quoted provision: (1) by using a fictitious name; (2) by falsely pretending to possess power, influence, qualifications, property, credit, agency, business or imaginary transactions; and (3) by means of other similar deceits. Under this class of estafa, the element of deceit is indispensable. In the present case, the deceit consists of accused-appellants false statement or fraudulent representation which was made prior to, or at least simultaneously with, the delivery of the money by the complainants. To convict for this type of crime, it is essential that the false statement or fraudulent representation constitutes the very cause or the only motive which induces the complainant to part with the thing of value.

Accused-appellants led private complainants to believe that they possessed the power, means and legal qualifications to provide the latter with work in Italy, when in fact they did not. Private complainants parted with their hard-earned money and suffered damage by reason of accused-appellants deceitful and illegal acts. The elements of deceit and damage for this form of estafa are indisputably present, hence their conviction for estafa was proper.

PEOPLE OF THE PHILIPPINES VS. ROMEO BANDIN G.R. NO. 176531, APRIL 24, 2009

RAPE

Facts: Accused-appellant Romeo Bandin was charged for the crime of rape against complainant-victim, AAA, a 16 year old woman, against her will. Under the facts, AAA woke up at past midnight because she felt a heavy burden on top of her. It was a naked man who was holding her tightly and who uttered in a commanding voice, "Dont move!" She recognized the mans voice as belonging to her brother-in-law, the appellant in this case. Thereafter, appellant removed the victims short pants and underwear. AAA covered her genitals with her right hand and pleaded with her brother-in-law to stop. Appellant, however, proceeded to remove AAAs hand from her genitals, spread her legs and immediately inserted his penis inside her vagina. She shouted for help several times but no one responded. She was too frightened to resist appellant because he was armed with a long firearm which he placed beside her. Appellants defense hinged on denial and alibi. He contended that he did not rape AAA. He claimed that he was sleeping in the bunker of the CAFGU station in Tagpangi, Cagayan de Oro City, which was about two kilometers away from AAA's house. His testimony was corroborated by Encoy and Baal. The trial court found appellant guilty beyond reasonable doubt of the crime charged. The CA affirmed the RTC decision with modifications. It held that since the complaint contained no allegations pertaining to the aggravating circumstances of dwelling and use of deadly weapon, the same cannot be appreciated in the imposition of the penalty.

Issue: Did the trial court erred in convicting appellant for the crime of rape?

Ruling: No. The Supreme Court finds no reason to overturn the conclusion arrived at by the trial court as affirmed by the CA. It held that AAA's testimony was credible as she delivered her testimony in a clear, direct and positive manner. Through his voice, she positively identified appellant as the man who sexually abused her. Identification of an accused by his voice has been accepted, particularly in cases where, as in this case, the victim has known the perpetrator for a long time.

Consequently, appellant's defense of denial and alibi must crumble in the face of AAA's positive and clear identification of him as the perpetrator of the crime. Denial and alibi cannot be given greater evidentiary value than the testimonies of credible witnesses who testify on affirmative matters. Positive identification destroys the defense of alibi and renders it impotent, especially where such identification is credible and categorical.

People of the Philippines vs. Rolly Canares y Almanares

G.R. No. 174065, February 18, 2009


Statutory Rape

Facts:

The accused was a helper in the victims grandmothers house. When the victim was only about 9 or 10 years old, the accused, while she was sleeping undressed her, pulled her shorts and panty and had sexual intercourse with her. This was

repeated on several occasions but the victim can no longer recall the exact dates. An information for rape under RA 7610 was filed against the accused.

Issue:

Was statutory rape committed?

Held:

Yes. Statutory rape is committed by sexual intercourse with a woman below 12 years of age regardless of her consent to the act or lack of it.55 Proof of force, intimidation or consent is unnecessary; force is not an element of statutoryrape and the absence of free consent is conclusively presumed when the complainant is below the age of twelve.56 The law presumes that a woman below this age does not possess discernment and is incapable of giving intelligent consent to the sexual act.57 To convict an accused of the crime of statutory rape, the prosecution must prove: first, the age of the complainant; second, the identity of the accused; and last but not the least, the carnal knowledge between the accused and the complainant.58 The first and second elements have been established by the presentation of a Certification from the Office of the Municipal Civil Registrar of Silang, Cavite dated April 21, 1999 stating that VICTIM was born on September 8, 1982.59Hence, she was only 9, or at most 10, years old when the rape was committed in 1992. In and out of court, she consistently identified Canares as her rapist.60

Carnal knowledge is proven by proof of the entry or introduction of the male organ into the female organ; the "touching" or "entry" of the penis into the labia majora or the labia minora of the pudendum of the victims genitalia constitutes consummated rape.61 The prosecution proved this element when VICTIM narrated during the trial the details of her rape, committed sometime in 1992. VICTIMs testimony is supported by physical and supporting testimonial evidence. There was the healed laceration found in her hymen which is remarkably compatible with her claim of sexual molestation. Dr. Madrid, in testifying on the healed laceration, stated that it could have been caused by a penis.

People of the Philippines vs. Alberto Buban G.R. No. 172710, October 30, 2009 Statutory Rape Facts: The victim was an orphaned minor who lived in her cousins house after her parents died. In four occasions, she was forced by the accused to have carnal knowledge with him. As a result, four informations of rape were filed against him. The accused defended himself that there was no rape because according to him, they were in fact lovers. He was unable, however, to show any love letter or picture which would prove that fact. After trial of the four informations, the trial court found the accused guilty on all four counts. On appeal, the accused interposed a number of defenses. Accused-appellant puts at issue the credibility of VICTIM, specifically as regards the third rape which occurred on January 29, 1996. He avers that it is contrary to human nature and experience that after having been previously raped twice, VICTIM would still feel comfortable, in the presence of appellant, as she was able to speak to him casually as if nothing traumatic happened between them and she even managed to sleep in her room without locking its door while accused-appellant was in the

sala watching television. Accused-appellant also relies on the inconsistencies between VICTIMs testimony as to the date of the commission of the four (4) rape incidents and as stated in the four (4) Informations. He alleged that on direct examination, VICTIM declared that she was sexually abused on October 12, 1995, November 15, 1995, November 24, 1995 and January 29, 1996, while the four (4) Informations clearly stated that the rape incidents took place on October 12, 1995, November 15, 1995, January 29, 1996 and the last one on February 24, 1996. Accused-appellant further asserts that there could be no rape where the sexual act was consensual. He maintains that if the sexual intercourse was truly against VICTIMs will, she could have easily cried for help when he was pulling her on the bed and she could have prevented the second rape by not going in the room knowing fully well that she was alone with the accused-appellant at that time Issue: Was rape committed? Held: In reviewing rape cases, the Court is guided by the following: (a) an accusation of rape can be made with facility; it is difficult to prove but more difficult for the person accused, though innocent, to disprove; (b) due to the nature of the crime of rape in which only two persons are usually involved, the testimony of complainant must be scrutinized with extreme caution; and (c) the evidence for the prosecution must stand or fall on its own merits and cannot be allowed to draw strength from the weakness of the evidence for the defense. The credibility of the offended party is crucial in determining the guilt of a person accused of rape. By the very nature of this crime, it is usually only the victim who can testify as to its occurrence. Thus, in rape cases, the accused may be convicted solely on the basis of the testimony of the victim, provided that such testimony is credible, natural, convincing and consistent with human nature and the

normal course of things. Else wise stated, the lone testimony of the offended party, if credible, suffices to warrant a conviction for rape. Guided by these judicial doctrines, the Court scrutinized all the pieces of evidence on record, especially the testimony of VICTIM and we find no reason to overturn the trial courts assessment of her credibility, which had the opportunity of observing VICTIMs manner and demeanor on the witness stand. VICTIMs testimony was indeed candid, spontaneous and consistent. just as the trial court observed. Well-established is the rule that for the crime of rape to exist, it is not necessary that the force employed be so great or be of such character that it could not be resisted; it is only necessary that the force employed by the guilty party be sufficient to consummate the purpose for which it was inflicted. In other words, force as an element of rape need not be irresistible; as long as it brings about the desired result, all considerations of whether it was more or less irresistible are beside the point. Intimidation must be viewed in the light of the perception of the victim at the time of the commission of the crime, not by any hard and fast rule; it is therefore enough that it produced fear -fear that if she did not yield to the bestial demands of her ravisher, some evil would happen to her at that moment or even thereafter. In the present case, there can be no doubt that accused-appellant employed that amount of force sufficient to consummate rape. At the time rape incidents took place, the victim was only seventeen (17) years old, while accusedappellant was more or less twenty-seven years old and in his prime. The obvious disparity between their physical strengths manifests the futility of any resistance. We also reject the argument of accused-appellant that it is simply contrary to human nature and experience for VICTIM who, after having been previously ravished twice, remained calm talking to him and even slept in her room without locking its door, knowing fully well that he was still in the living room watching television. VICTIM reasoned out that she did not lock her room because her aunt, who was then watching betamax movie in their neighbor, is going to sleep in that room. Accused-appellant failed to show any ill motive, on the part of the victim to fabricate such a story. The testimony of accused-appellant that the reason for the filing

of these charges against him was because of the quarrel between her wife and VICTIM after the former learned about his illicit relationship with VICTIM is opposed to what he declared in open court that the only time his wife came to know about their relationship was after the complaints were filed because he told her about it when he was already in jail. Since there is no evidence to show any improper motive on the part of the complainant to testify against the accused or to falsely implicate him in the commission of a crime, the logical conclusion is that the testimony is worthy of full faith and credence. As regards the alleged discrepancies as to the dates of the commission of the rape, the rule is well settled that in rape cases, the date or time of the incident is not an essential element of the offense and therefore need not be accurately stated People of the Philippines vs. Ignacio Poras G.R. No. 177747, Feb. 16, 2010 Rape lowered to Acts of Lasciviousness

Facts:

The victim was made to drink coffee (which was drugged) by the accused which caused her to sleep. When she woke up, she saw the accused moving on top of her and touching her private parts. She also noticed that the strap of her bra had been removed, and her panty already lowered to her knees. When she pushed the appellant, the latter raised his brief and went to his room, threatening to kill her if she would disclose the incident to anyone. She did not call for help because she felt weak. On the witness stand, she also said that she felt pain in her vagina. The trial court found the accused guilty of rape.

Issue:

Do the facts show beyond reasonable doubt that the crime of rape was committed?

Held:

No. We find that the prosecution failed to prove the appellants guilt beyond reasonable doubt of the crime of rape. We convict him instead of the lesser of acts of lasciviousness, included in rape, as the evidence on record shows the presence of all the elements of this crime. In the present case, the lower courts convicted the appellant of rape based on the following circumstances: (a) the appellant made VICTIM drink coffee which made her fall asleep; (b) VICTIM saw the appellant lying beside her, moving on top of her, and touching her private parts when she woke up; (c) VICTIMs panty had been lowered to her knees, and the strap of her bra had been removed; (d) the appellant put on his briefs and shorts after VICTIM pushed her; (e) VICTIM felt pain in her private parts, and saw blood stains on her panty; (f) the appellant threatened to kill VICTIM if she disclosed the incident to anyone; and (g) the examining physician found deep-healed lacerations in VICTIMs vagina.

After due consideration of the evidence adduced, that the circumstantial evidence failed to clearly establish an unbroken chain leading to the fair and reasonable conclusion that the appellant raped VICTIM.

We cannot equate a ruptured hymen with rape. medical certificate or the testimony of the physician is presented not to prove that the victim was raped but to show that the latter had lost her virginity. Consequently, standing alone, a physicians finding that the hymen of the alleged victim was lacerated does not prove rape. It is only when this is corroborated by other evidence proving carnal knowledge that rape may be deemed to have been established.

Thus, while the healed lacerations are undisputed, they can only prove, in the absence of any other evidence, that VICTIM has had prior sexual experience. Specific proof of penile contact, on or about the time the appellant allegedly raped her, is missing.

Even assuming, for the sake of argument, that the appellant succeeded in inserting his fingers in VICTIMs vagina, this act still would not suffice to convict the appellant of rape. In 1994, the insertion of one or more fingers into a womans vagina without her consent did not constitute rape. It was only in 1997 that the law on rape was expanded to include this act.

We find it highly unlikely that the appellant inserted his penis into VICTIMs vagina while the latters panty was lowered to her knees. Common sense and ordinary human experience show that penile penetration is extremely difficult, if not almost impossible under this situation, unless the victims legs were spread apart.

Considering that VICTIM was an unmarried 13-year old, she would have been in unusually deep sleep in order not to feel the pain and sensation reasonably expected from the insertion of a penis into her young, vaginal canal. We are baffled how could

she have slept through a consummated sexual intercourse and awakened only after its completion. The conviction in a rape case though must rest on evidence, not on mere possibility.

We cannot equate VICTIMs testimony of pain in her private parts with rape. Carnal knowledge, not pain, is the element of consummated rape and we believe that it would be a dangerous proposition to equate a victims testimony of pain, in the absence of any other evidence, with carnal knowledge. The peril lies in the facility of asserting pain. Pain, too, can come from various reasons other than carnal knowledge; it is also subjective and is easy to feign.

Finally, we cannot help but observe that VICTIM, in her direct testimony, revealed that she merely came to the conclusion that the appellant had raped her afterbeing told by the examining physician that the result of the medical examination was positive, and that something had happened to her.

In view of the foregoing, the findings of conviction is vacated. Accused is instead found guilty of crime of acts of lasciviousness. People of the Philippines vs. Felix Ortoa y Obia G.R. No. February 23, 2009 Rape and Acts of Lasciviousness

Facts:

The accused has two daughters, Victim 1 and Victim 2. When Victim 1 was only three years (3) old, the accused would make her lie down and insert his finger into her vagina until she cries. As she grew older, these incidents were repeated a number of times. When Victim 1 turned the age of six (6) the accused started to have sexual intercourse with her by undressing her and inserting his penis into her vagina, making push and pull movements until he ejaculates. The last time the accused did this was when Victim 1 was already 13 years old. Insofar as Victim 2 is concerned, the accused started to have sexual encounter with her when she was already eight (8) years old. In one instance, the accused partially inserted his penis into the labia of her vagina and in another instance, fully penetrated her. On appeal, the accused contended among many others that it is inconceivable for the Victims to not report the crimes immediately after each incident considering that as children the natural reaction would have to report it immediately to a relative. Furthermore, he argued that the physical examination of the victims have shown that their hymen are still intact.

Issue:

Were the two counts of rape and acts of lasciviousness proven beyond reasonable doubt under the facts of the case?

Held:

Yes. The Court is not persuaded by appellant's arguments that it is inconceivable for VICTIM to only report her rape and molestation to the authorities when she was already 13 years old, considering that she claimed that appellant started to

sexually assault her when she was only 3 years old; that her natural reaction would be to tell her ordeal to her mother right away; that if complainants really wanted to protect themselves, it was uncharacteristic for them not to tell their molestation to anyone as there was no threat to their lives, nor was there anything that would have prevented them from divulging their sufferings. Silence and delay in reporting the crime of rape have not always been construed as indications of a false accusation.44 This principle applies with greater force where, as in this case, the victims were of tender age at the time of the rape incidents and were therefore susceptible to intimidation and threats of physical harm, especially from a close relative.

The Court is not persuaded by appellant's contention that VICTIM was never sexually abused because the medico-legal findings showed that she was still in a virgin state when she was examined. The medico-legal expert who examined VICTIM testified that it was possible for a male organ to penetrate the labia minora and leave the hymen still intact. A freshly broken hymen is not an essential element of rape.53 Even the fact that the hymen of the victim was still intact does not rule out the possibility of rape.54Research in medicine even points out that negative findings are of no significance, since the hymen may not be torn despite repeated coitus. With respect to the crime of acts of lasciviousness, it is settled that the lone testimony of the offended party, if credible, is sufficient to establish the guilt of the accused.63 Such are the testimonies of victims who are young, immature, and have no motive to falsely testify against the accused, as in the instant case. Against the overwhelming evidence of the prosecution, appellant merely interposed the defense of denial. Categorical and consistent positive identification, absent any showing of ill motive on the part of the eyewitness testifying on the matter, prevails over the defense of denial. PO3 Benito Sombilon vs. People of the Philippines

G.R. No. 175528, September 30, 2009 Acts of Lasciviousness

Fact:

The victim was brought to the police station for investigation in connection with a complaint for theft. To extract a confession from the victim (who was 15 years old), the accused, who was a police officer, brought her inside one of the rooms in the police station, pointed a gun at her; she was interrogated and electrocuted. Not satisfied, the police officer asked her: Dalaga ka na ba? And was told by the officer: I am single too. Thereafter, the police officer touched different parts of her body, mashed her breast, kissed her in the cheek and touched her private parts. An information for acts of lasciviousness was filed against the police officer. She was found by the trial court guilty as charged. In handing its judgment, the trial court, despite its absence in the information, appreciated the aggravating circumstance of taking advantage of public position. On appeal the accused contended that there was no lewdness in his acts hence, the best that can imposed upon him is the crime of unjust vexation and not acts of lasciviousness. Further he contended that since the aggravating circumstance of public position was not alleged, the same must not be appreciated against him.

Issues: 1) Were the trial and the appellate courts correct in convicting the accused? 2) Were the trial and appellate courts correct in appreciating the aggravating circumstance of public position?

Held:

On the first issue, the trial and appellate courts were correct in finding the accused guilty of the crime charged. For an accused to be convicted of acts of lasciviousness under the foregoing provision, the prosecution is burdened to prove the confluence of the following essential elements: (1) that the offender commits any act of lasciviousness or lewdness; and (2) that it is done under any of the following circumstances: (a) by using force or intimidation; (b) when the offended woman is deprived of reason or otherwise unconscious; or (c) when the offended party is under twelve (12) years of age. The term "lewd" is commonly defined as something indecent or obscene; it is characterized by or intended to excite crude sexual desire. That an accused is entertaining a lewd or unchaste design is necessarily a mental process the existence of which can be inferred by overt acts carrying out such intention, i.e., by conduct that can only be interpreted as lewd or lascivious. The presence or absence of lewd designs is inferred from the nature of the acts themselves and the environmental circumstances.

Undoubtedly, petitioner committed acts which fall within the above described lascivious conduct. It cannot be viewed as mere unjust vexation as petitioner would have the Court do. The intention of petitioner was intended neither to merely annoy or irritate the victim nor to force her to confess the theft. He could have easily achieved that when he electrocuted the latter. Petitioner intended to gratify his sexual desires. Undeniably, appellant committed lewd acts against AAA. "Lewd" is defined as obscene, lustful, indecent, and lecherous. The evidence shows that appellant committed lewd acts against AAA when he touched her "all over her body" which includes mashing her breasts, touching her private parts, and kissing her on the cheek. These acts were clearly done with lewd designs as appellant even

previously asked AAA, as if it was a prelude for things to come, "Dalaga ka na ba?" and thereafter conveyed to her that "he is single too."14 Moreover, appellant employed force and intimidation when he committed these acts on AAA. In fact, as found by the trial court, appellant pointed a gun at the forehead of AAA as evidenced by the bruises on her forehead. Further, the medical Certificate shows that AAA suffered slight physical injuries which include "multiple slight contusion of bilateral breast areas" which supports AAAs claim. Petitioners assertion that the locus criminis i.e., the police station makes it unlikely for him to commit the crime of acts of lasciviousness is specious. The presence of other policemen on duty and of the victims mother outside the room where the incident took place does not render commission of the offense impossible As to the appreciation of the aggravating circumstance of taking advantage of public position, petitioner points out that said circumstance was not alleged in the information. It is now a requirement that the aggravating as well as the qualifying circumstances be expressly and specifically alleged in the complaint or information. Otherwise, they cannot be considered by the trial court in its judgment, even, if they are subsequently proved during trial.19 A reading of the Information shows that there was no allegation of any aggravating circumstance. Judgement was therefore affirmed with modification of the penalty by eliminating the appreciation of the above-discussed aggravating circumstance. Victoria Jarillo vs. People of the Philippines G.R. No. 164435, June 29, 2010 Bigamy

Facts:

The petitioner was previously convicted of bigamy which was affirmed all the way up to the Supreme Court. In her motion for reconsideration, she posits that since petitioner's marriages were entered into before the effectivity of the Family Code, then the applicable law is Section 29 of the Marriage Law (Act 3613), instead of Article 40 of the Family Code, which requires a final judgment declaring the previous marriage void before a person may contract a subsequent marriage.

Issue: Should her conviction be reversed?

Held:

No. which is a rule of procedure, should be applied retroactively because Article 256 of the Family Code itself provides that said "Code shall have retroactive effect insofar as it does not prejudice or impair vested or acquired rights." The Court went on to explain, thus: The fact that procedural statutes may somehow affect the litigants' rights may not preclude their retroactive application to pending actions. The retroactive application of procedural laws is not violative of any right of a person who may feel that he is adversely affected. The reason is that as a general rule, no vested right may attach to, nor arise from, procedural laws.4

In Marbella-Bobis v. Bobis,5 the Court pointed out the danger of not enforcing the provisions of Article 40 of the Family Code, to wit: In the case at bar, respondent's clear intent is to obtain a judicial declaration of nullity of his first marriage and thereafter to invoke that very same judgment to prevent his prosecution for bigamy. He cannot have his cake and eat it too. Otherwise, all that an adventurous bigamist has to do is disregard Article 40 of the Family Code, contract a subsequent marriage and escape a bigamy charge by simply claiming that the first marriage is void and that the subsequent marriage is equally void for lack of a prior judicial declaration of nullity of the first. A party may even enter into a marriage aware of the absence of a requisite usually the marriage license and thereafter contract a subsequent marriage without obtaining a declaration of nullity of the first on the assumption that the first marriage is void. Such scenario would render nugatory the provision on bigamy. x x x lawphil The foregoing scenario is what petitioner seeks to obtain in her case, and this, the Court shall never sanction. Clearly, therefore, petitioner's asseveration, that Article 40 of the Family Code should not be applied to her case, cannot be upheld.

ICKY MOSTER VS PEOPLE OF THE PHILPPINES G.R. No. 167461, February 19, 2008 BP 22

Facts: Petitioner obtained from Presas a loan of P450,000, for which the petitioner issued as payment three postdated PhilBank checks. The three checks were all payable to cash. Presas testified she did not deposit

the checks on their due dates upon petitioners request and assurance that they would be replaced with cash. When she could not wait any longer, Presas deposited Check Nos. 026138 and 026124 in her Westmont Bank account, only to be notified later that the checks were dishonored because the account had been closed. Presas said she did not deposit Check No. 026137 after she agreed to petitioners request to withhold its deposit as it had not yet been funded. After receiving notice that Check Nos. 026138 and 026124 had been dishonored, Presas immediately informed petitioner thereof and demanded payment for the value of the checks. This demand, however, went unheeded. In a letter, Presas through counsel, demanded from petitioner the settlement of P367,602, representing the total value of the three checks, within five days from receipt. Petitioner, however, did not comply. Thus, three Informations for violation of B.P. Blg. 22 were filed against petitioner.

Issue: Is petitioner guilty of a violation of BP 22?

Ruling: NO. B.P. Blg. 22 punishes as malum prohibitum the mere issuance of a worthless check, provided the other elements of the offense are proved. Section 1 enumerates the elements of B.P. Blg. 22, as follows: (1) the making, drawing, and issuance of any check to apply on account or for value; (2) the knowledge of the maker, drawer, or issuer that at the time of issue he

does not have sufficient funds in or credit with the drawee bank for the payment of the check in full upon its presentment; and (3) the subsequent dishonor of the check by the drawee bank for insufficiency of funds or credit or dishonor for the same reason had not the drawer, without any valid cause, ordered the bank to stop payment. Upon careful examination of the records, however, the Court found that only the first and third elements have been established by the prosecution. By her own admission, petitioner issued the three subject checks, two of which were presented to PhilBank but were dishonored and stamped for the reason Account Closed. Under Section 3 of B.P. Blg. 22, the introduction in evidence of the dishonored check, having the drawees refusal to pay stamped or written thereon, or attached thereto, with the reason therefor as aforesaid shall be prima facie evidence of the making or issuing of the said checks and the due presentment to the drawee for payment and the dishonor thereof, and that the same was properly dishonored for the reason written, stamped or attached thereto by the drawee on such dishonored checks. As to the second element, Section 2 of B.P. Blg. 22 creates the presumption that the issuer of the check was aware of the insufficiency of funds when he issued a check and the bank dishonored it. This presumption, however, arises only after it is proved that the issuer had received a written notice of dishonor and that, within five days from receipt thereof, he failed to pay the amount of the check or to make arrangements for its payment.

Ordinarily, preponderance of evidence is sufficient to prove notice. But in criminal cases, the quantum of proof required is proof beyond reasonable doubt. In the instant case, the prosecution merely presented a copy of the demand letter allegedly sent to petitioner through registered mail and the registry return card. There was no attempt to authenticate or identify the signature on the registry return card. All that we have on record is an illegible signature on the registry receipt as evidence that someone received the letter. As to whether this signature is that of petitioner or her authorized agent remains a mystery. Receipts for registered letters and return receipts do not by themselves prove receipt; they must be properly authenticated to serve as proof of receipt of the letters, claimed to be a notice of dishonor. Unfortunately, the prosecution presented only the testimony of Presas to prove mailing and receipt of the demand letter

Potrebbero piacerti anche